Está en la página 1de 84

BANCO CENTRAL DE RESERVA DEL PERÚ

CURSO DE ACTUALIZACION Y SELECCION

BANCO DE EXAMENES 2007 - 2014

Recomendaciones:
1. El examen consta de examen de macroeconomía, microeconomía e
inglés. EI tiempo asignado para el desarrollo del examen es de 2
HORAS

2. Marque un círculo con lapicero alrededor de la opción que considere


correcta en cada respuesta. No marque más de una opción, porque
en el caso de marcar más de una opción, sólo se considerará Ia
respuesta incorrecta

3. La respuesta correcta vale 5 puntos, la respuesta incorrecta tiene va-


lor negativo de 1 punto. La pregunta no respondida no da puntaje
EXAMEN DE ACTUALIZACION Y SELECCIÓN BCRP
COMPILADOR MARINO ENEQUE

EXAMEN DE ACTUALIZACIÓN Y SELECCIÓN 2014

MACROECONOMÍA 2014

1. La equivalencia Ricardiana implica 0,3 y, además, la tasa de creci-


que: miento poblacional y la de deprecia-
ción viene dadas por n = 0,05 y δ =
a) La política fiscal no afecta el creci-
miento de largo plazo 0,1. La tasa de crecimiento de aho-
rro es de 20%. Se pide determinar
b) Un incremento del gasto público ge-
nera expectativas de futuros aumen- qué afirmaciones son consistentes
tos en las tasas impositivas con los valores proporcionales.
c) Un mayor gasto público, genera un a) En estado estacionario, el ratio pro-
aumento de la misma magnitud en el ducción total-capital total tiene un
ahorro privado valor de 0,75
d) Todas las anteriores b) La economía tarda aproximada-
e) Ninguna de las anteriores mente 6,6 periodos en alcanzar la
2. Señale cuál de las siguientes afir- mitad de su trayectoria al estado es-
tacionario
maciones no se cumple en el mo-
c) El nivel de consumo por trabajador
delo de Solow-Swan:
determinado por la regla de oro de
a) El equilibrio del modelo es global- Phelps es de 1,1
mente estable; es decir, el nivel de d) Solo la respuestas b y c son correc-
capital por trabajador siempre con- tas
verge al estado estacionario sin im- e) Ninguna de las anteriores
portar el nivel inicial del capital
4. Señale la afirmación correcta sobre
b) En ausencia de progreso tecnoló-
gico el PBI per cápita no crece en es- el modelo de Ramsey-Koopmans:
tado estacionario a) A través de la optimización de los
c) Las economías crecen más rápido agentes se logra endogenizar las
cuanto se encuentra más cerca a fuentes de crecimiento de la econo-
sus estados estacionarios mía, en contraste al modelo de So-
d) Los países más ricos son aquellos low
que intervienen más y experimentan b) La tasa de ahorro no es una variable
menores tasas de crecimiento de- exógena
mográfico c) La convergencia al estado estacio-
e) Ninguna de las anteriores nario está asegurada por los su-
3. Sea una economía se Solow sin puestos del modelo
crecimiento tecnológico donde la d) La ecuación de Euler resultante de la
optimización de los agentes deter-
producción pre cápita viene deter-
mina la senda de crecimiento del ca-

minada por y (t )  k (t ) con α = pital
e) Ninguna de las anteriores

2 de 84
EXAMEN DE ACTUALIZACION Y SELECCIÓN BCRP
COMPILADOR MARINO ENEQUE

5. Sobre la acumulación de capital en 8. ¿Cuál es la relación entre circulante


los modelos de Solow y Ramsey se y la emisión primaria?
puede decir que:
a) La emisión primaria es siempre igual
a) La regla de oro se deriva de la maxi- que el circulante
mización de la utilidad en el modelo b) El circulante es siempre mayor que
de Solow la emisión primaria
b) La región donde el capital por traba- c) El circulante es parte de la emisión
jador es mayor al nivel de capital de primaria
oro por trabajador es denominada d) La base monetaria es parte del cir-
región dinámicamente ineficiente culante
c) La regla de oro modificada se deriva e) Ninguna de las anteriores
de la maximización del consumo en
el modelo de Ramsey 9. En la teoría de paridad no cubierta
d) Un incremento imprevisto del gasto de tasas de interés, ante un incre-
de gobierno no produce efecto mento de la tasa de interés domés-
“crowding out” tica, se cumple:
e) Ninguna de las anteriores
a) Se generan expectativas de aprecia-
6. Sobre los modelos de crecimiento ción
económico endógeno no es cierto b) Se generan expectativas de depre-
que: ciación
c) El tipo de cambio spot se aumenta
a) El motor de crecimiento de la econo-
mía viene explicado dentro del mo- d) El tipo de cambio futuro aumenta
delo e) Ninguna de las anteriores
b) La tasa de crecimiento del capital 10. En una economía con tipo de cam-
por trabajador puede ser positiva en bio fijo se cumple que:
estado estacionario
c) Hay ausencia de rendimientos de- a) La tasa de interés doméstica sólo
crecientes a escala cambia si la tasa de interés externa
d) El papel del capital humano juega un cambia.
rol importante b) La cantidad de dinero que circula do-
e) Ninguna de las anteriores mésticamente, la determina la ga-
nancia de reservas internacionales.
7. El multiplicador bancario será ma- c) Los agentes no pueden formar ex-
yor cuando: pectativas sobre la inflación.
a) La preferencia por circulante au- d) Opciones a y b son ciertas.
mente. e) Ninguna de las anteriores
b) La tasa de encaje se reduzca. 11. Una depreciación del tipo de cam-
c) La preferencia por depósitos se re- bio real:
duzca.
d) El crédito bancario se reduzca. a) Favorece a las exportaciones y des-
favorece a las importaciones
e) Ninguna de las anteriores

3 de 84
EXAMEN DE ACTUALIZACION Y SELECCIÓN BCRP
COMPILADOR MARINO ENEQUE

b) Desfavorece a las expectativas y fa- a) Si se cumple la PPC, entonces se


vorece a las importaciones cumple la ley de un solo precio
c) Favorece a las exportaciones e im- b) La PPC absoluta nos dice que la de-
portaciones preciación del tipo de cambio entre
d) Desfavorece a las exportaciones e dos países se iguala a la diferencia
importaciones entre sus inflaciones en un determi-
e) Ninguna de las anteriores nado periodo de tiempo
c) Si PPC relativa se cumple, entonces
12. En el Modelo Ricardiano de Comer- el tipo de cambio real se mantiene
cio Internacional se concluye que: constante
d) Si la PPC absoluta se cumple, en-
a) Cada país se especializa en el pro-
tonces el tipo de cambio real es dis-
ducción del bien en el que tiene rela-
tinto de 1
tivamente mayor requerimiento de
mano de obra e) Ninguna de las anteriores
b) Cada país se especializa en el pro- 15. Una intervención esterilizada en el
ducción del bien en el que tiene me- mercado cambiario por parte de
nor requerimiento de mano de obra Banco Central implica:
c) Cada país se especializa en el pro-
ducción del bien en el que tiene ma- a) Solo cambios en los pasivos del ba-
yor requerimiento de mano de obra lance del Banco Central
d) Cada país se especializa en el pro- b) Cambios en los activos y pasivos del
ducción del bien en el que tiene rela- balance del Banco Central
tivamente menor requerimiento de c) Solo cambios en los activos del ba-
mano de obra lance del Banco Central
e) Ninguna de las anteriores d) Ningún cambio en los activos o pasi-
13. Si se cumple la paridad de tasas de vos del balance del Banco Central
interés no cubierta y la tasa de inte- e) Ninguna de las anteriores
rés doméstica 4% y la tasa de inte- 16. Respecto a la Balanza de Pagos
rés extranjera es de 1%. Marque la del Perú no es cierto que:
alternativa correcta:
a) Las transacciones de exportaciones
a) Se espera una depreciación del tipo e importaciones de bienes y servi-
de cambio nominal 5%. cios se registran en la cuenta co-
b) Se espera una depreciación del tipo rriente.
de cambio nominal 3% b) La renta de factores se encuentra
c) Se espera una apreciación del tipo dentro de la cuenta financiera.
de cambio nominal 3% c) Si omitimos los errores y omisiones
d) Se espera una apreciación del tipo netas y el efecto valuación, entonces
de cambio nominal 5% la suma de los resultados de la
cuenta corriente, cuenta financiera y
e) Ninguna de las anteriores
financiamiento excepcional es la va-
14. Respecto a la paridad del poder de riación en las reservas internaciona-
compra (PPC) es cierto que: les netas

4 de 84
EXAMEN DE ACTUALIZACION Y SELECCIÓN BCRP
COMPILADOR MARINO ENEQUE

d) Las transacciones corrientes se en- e) Ninguna de las anteriores


cuentren dentro de la cuenta co-
rriente 19. En el modelo IS-LM se cumple que:
e) Ninguna de las anteriores a) Un incremento del gasto fiscal ge-
nera mayores tasas de interés
17. En una economía pequeña y
abierta, un incremento transitorio b) Una contracción de la cantidad de di-
nero reduce el producto
de la productividad genera:
c) El producto aumenta si las familias
a) Superávit en cuenta corriente prefieren ahorrar más
b) Déficit en cuenta corriente d) Las opciones a y b son ciertas
c) Reducción en la posición neta de ac- e) Ninguna de las anteriores
tivos domésticos
20. La política de expansión cuantita-
d) Aumento en la posición neta de pa- tiva de la FED se utilizó para:
sivos domésticos
e) Ninguna de las anteriores a) Reducir las tasas de interés de corto
plazo en el mercado de dinero de los
18. La inflación aumenta siempre que: EE.UU.
a) Las expectativas de inflación au- b) Reducir las tasas de interés de corto
menten. plazo en economías emergentes.
b) El tipo de cambio se incrementan de c) Reducir las tasas de interés de largo
manera persistente. plazo en los EE.UU.
c) La productividad de las empresas se d) Elevar los precios de los commodi-
reduzca persistentemente. ties.
d) Todas las anteriores. e) Ninguna de las anteriores

MICROECONOMÍA 2014

21. Un consumidor dispone de 12000 d) El precio de las películas en térmi-


euros al mes para gastar en ver pe- nos de los demás bienes es 0.20
lículas (bien x) y en el resto de los e) Ninguna de las anteriores
bienes (bien y), cuyos precios son 22. Un gimnasio ofrece únicamente
respectivamente Px = 500 y Py = clases de ballet y de gimnasia. El
100. ¿Cuál de las siguientes afirma- departamento de marketing decide
ciones es verdadera? regalar un bono de 21 horas men-
a) El número máximo de películas que suales a cada uno de los 20 resi-
puede consumir es 30 dentes del barrio donde está ubi-
b) Si se consume 10 películas, se po- cado el gimnasio que se presenten
dría consumir 80 de los demás bie- primero para conocer las instalacio-
nes nes. La duración de las clases es
c) El precio de las películas en térmi- de 1 hora y 30 minutos para las cla-
nos de los demás bienes son 5 ses de ballet, y de 45 minutos para

5 de 84
EXAMEN DE ACTUALIZACION Y SELECCIÓN BCRP
COMPILADOR MARINO ENEQUE

las de gimnasia. La ordenada en el 25. Considere un consumidor con una


origen y la pendiente de la restric- función de utilidad entre los bienes
ción presupuestaria son respectiva- X e Y de la forma: U = X2Y2. Este
mente: consumidor tiene ingresos por 200
euros y se enfrenta a unos precios
a) 21; -1.5
Px = 10, Py = 20. Ante la posibilidad
b) 28; -2
que le ofrecen de pagar una cuota
c) 25; 1.5
de 20 euros que le da derecho a 4
d) La restricción presupuestaria solo se
unidades del bien Y, por encima de
puede expresar en unidades mone-
tarias, no en horas. las cuales se paga el precio de mer-
cado, el consumidor;
e) Ninguna de las anteriores.
23. Considere un consumidor con pre- a) Se mostrara indiferente entre acep-
tar o no dicha posibilidad
ferencias estrictamente convexas.
El valor absoluto de la pendiente de b) Ganará con la posibilidad que le
ofrecen
una curva de indiferencia en el
c) Perderá con la posibilidad que le
punto (X=3, Y=4) es 2. ¿Cuánto
ofrecen
vale el valor absoluto de dicha pen-
d) Podrá tanto ganar como perder, ya
diente cuando Y=2? que no tenemos datos suficientes
a) Mayor que 2 para saberlo
b) Menor que 2 e) Ninguna de las anteriores
c) No se puede asegurar sin conocer el 26. Suponga que la curva de demanda
valor de X de un bien es elástica. Si el precio
d) No se puede asegurar nada sin co- del bien disminuye;
nocer la función de la utilidad
e) Ninguna de las anteriores a) Disminuirá el gasto del consumidor
de ese bien
24. Mario siempre acude a la feria del b) Aumentará el precio de un bien sus-
libro a comprar las últimas noveda- tituto
des. Sus preferencias entre nove- c) Aumentará el precio de un bien com-
las (bien X) y ensayos (bien Y) son plementario
tales que siempre lee, como mí- d) Aumentará el gasto del consumidor
nimo, dos novelas por cada ensayo. en ese bien
Las preferencias de Mario vienen e) Ninguna de las anteriores
representadas por una función: 27. Se tiene un bien Giffen cuando ante
a) U = X + 0,5Y un cambio en su respectivo precio.
b) U = min (X,2Y) a) El efecto ingreso va en mismo sen-
c) U = min (2X,Y) tido y es mayor al efecto sustitución
d) U=X1/2 +Y b) El efecto ingreso va en mismo sen-
e) Ninguna de las anteriores. tido y es menor al efecto sustitución

6 de 84
EXAMEN DE ACTUALIZACION Y SELECCIÓN BCRP
COMPILADOR MARINO ENEQUE

c) El efecto ingreso va en sentido e) Ninguna de las anteriores


opuesto y es mayor al efecto sustitu-
ción 30. Dado el problema clásico del con-
sumidor, en la elección óptima se
d) El efecto ingreso va en sentido
opuesto y es menor al efecto sustitu- cumple que:
ción a) En el margen, la utilidad de cada
e) Ninguna de las anteriores bien debe ser la misma
28. En equilibrio general competitivo: b) En la margen, la utilidad de la última
unidad monetaria gastada en cada
a) Los beneficios de casa firma deben bien debe ser la misma
ser positivos, de lo contrario no hay c) La tasa marginal de sustitución entre
incentivos para producir cualquier par de bienes debe ser
b) Los factores son remunerados de igual a su precio relativo
acuerdo a su productividad d) Más de una es correcta
c) El valor agregado de los excesos de e) Ninguna de las anteriores
demanda no puede ser positivo
d) Más de una de las alternativas ante- 31. Considérese el modelo básico de
riores es correcta mercado, con una oferta creciente
e) Ninguna de las anteriores en precios y una demanda decre-
ciente en precios. Así, ante la apli-
29. En un modelo de duopolio con fir-
cación de un impuesto ad-valoren:
mas idénticas que enfrentan una
demanda común por el único a) Cuando la curva de oferta es perfec-
bien que producen y donde cada tamente elástica, los consumidores
firma fija el precio unitario de su experimentan la pérdida de bienes-
tar en su totalidad.
producto:
b) Cuando la curva de demanda es per-
a) En equilibrio de Nash, ambas firmas fectamente elástica, los productores
se coluden para fijar un precio mayor experimentan la pérdida de bienes-
a su coste marginal. tar en su totalidad.
b) En equilibrio de Nash, ambas firmas c) Cuando la curva de demanda es per-
se coluden para fijar un precio igual fectamente inelástica, los producto-
a su coste marginal. res experimentan la pérdida de bie-
c) En equilibrio de Nash, ambas firmas nestar en su totalidad.
fijan un precio mayor a su coste mar- d) Cuando la curva de demanda es per-
ginal, con la propiedad de que, a me- fectamente inelástica, los producto-
dida que el número de firmas au- res experimentan la pérdida de bie-
menta, el precio del equilibrio con- nestar en su totalidad
verge al coste marginal e) Todas las anteriores son correctas
d) En equilibrio de Nash, cada firma
iguala su precio a su coste marginal.

7 de 84
EXAMEN DE ACTUALIZACION Y SELECCIÓN BCRP
COMPILADOR MARINO ENEQUE

32. A continuación se presentan formas alternativas de producir 100 Tm. de


papa:

Trabajo Capital Tierra (Ha)


(horas) (horas)
1 105 30 10
2 500 10 15
3 300 20 18
4 120 30 18
5 250 20 15

¿Cuál(es) son formas de producción b) 3; 5


evidentemente ineficientes? c) 3; 4
d) 4
a) 1; 5 e) Ninguna de las anteriores

33. Dos comerciantes de bebidas alcohólicas tienen tres estrategias de propa-


ganda de su producto: resaltar las cualidades negativas del producto contra-
rio (A), no hacer propaganda (B), resaltar las cualidades positivas de su pro-
ducto (C). Los beneficios que se obtendrían para cada comerciante en todos
los casos posibles viene dado por la siguiente tabla de pagos:

Com. 2 A B C
Com. 1
A (1 , 1) (3, 0) (0 , 3)
B (0, 3) (2 , 2) (1, 2)
C (3, 0) (2 , 1) (2 , 2)

Se lee así: (x, y): Comerciante 1 recibe x y comerciante 2 recibe y.


Si el comerciante 1 no sabe qué a) B
estrategia tendrá el comerciante 2, b) C
¿Qué estrategia usaría el comer- c) B; C
ciante 1? También puede suceder d) A; C
que entre dos o las tres estrategias e) Ninguna de las anteriores.
no se puede distinguir que una sea
siempre mejor que otra

8 de 84
34. Se presenta a continuación un grupo de curvas de indiferencia: cada curva
representa las combinaciones de consumo A y B entre las que el individuo
tiene la misma utilidad. La flecha indica la dirección de preferencia del con-
sumidor: curva de indiferencia hacia la izquierda representa mayor utilidad
para el consumidor.

¿Qué afirmación es cierta respecto 35. Un consumidor tiene la siguiente


a las preferencias de este consu- función de utilidad sobre los pro-
midor? ductos x e y: U(x,y) = max [x-5, y-2]
Si el bien x cuesta 1 y el bien y
a) Menor consumo de B le da mayor
cuesta 2. ¿Qué afirmación es ver-
utilidad si A se mantiene constante
dadera?
b) Menor consumo de A le da menor
utilidad si B se mantiene constante a) En algún momento se podrá consu-
c) Mayor consumo de B le da menor mir ambos bienes a la vez
utilidad si A se mantiene constante b) Siempre se consumirá solo el bien x
d) c y b c) Siempre se consumirá solo el bien y
e) Ninguna de las anteriores d) Si se tiene diez soles se consumirá
5 unidades de x
e) Ninguna de las anteriores
EXAMEN DE ACTUALIZACION Y SELECCIÓN BCRP
COMPILADOR MARINO ENEQUE

ENGLISH

Narrow path central banks around the world are struggling to promote
growth without fomenting worrisome risk – taking (taken from the Econo-
mist, 21/06/2014)

Until the global financial crisis, central banks treated bubbles with benign neglect:
they were hard to detect and harder to deflate, so best left alone; the mess could
be mopped up after they burst. No self-respecting central bank admits to benign
neglect any longer. “No one wants to live through another financial crisis,” Janet
Yellen, then a candidate to head the Federal Reserve, said last year. “I would not
rule out using monetary policy as a tool to address asset-price misalignments.”

After six years of interest rates near zero the tension between central banks’ re-
sponsibility for output and inflation on one hand and financial stability on the other
is growing. On June 12th the Bank of England hinted it would pursue new
measures to curb ever-climbing property prices. Shortly afterwards Ms Yellen
fretted about the “reach for yield” and subdued volatility, a sign of investors’ com-
placency.

For Britain and America, the prospect of using interest rates to tackle financial
imbalances remains hypothetical. Not so in Norway and Sweden, where central
banks have been stingy with rate cuts for fear of inflating home prices and house-
hold debt. That has come at a cost: inflation is below target in both countries; in
Sweden it is negative. This could become entrenched: Andy Levin, an economist
at the IMF, recently noted that long-run inflation expectations in both countries
have also dropped below target.

In Sweden, the Riksbank’s stance has been deeply divisive. Two of the six mem-
bers of its executive board voted in April to cut the repo rate, which is now 0.75%.
Lars Svensson, an academic economist who left the board last year, calculates
that unnecessarily tight monetary policy since 1997 has raised unemployment by
0.8 percentage points. He believes it has also worsened Sweden’s imbalances
by slowing the growth of incomes more than the growth of debt, thereby raising
the household-debt ratio, now 174%.

Sweden’s parliament recently instigated an independent review of the Riksbank’s


policy. The central bank’s defenders argue that the consequences of a house-
hold-debt bust require pre-emptive action, and that tightening rules, such as
tougher underwriting standards for mortgages, while essential, is not enough. A
Riksbank staff study contradicts Mr Svensson’s findings, concluding that it is

10 de 84
EXAMEN DE ACTUALIZACION Y SELECCIÓN BCRP
COMPILADOR MARINO ENEQUE

lower rates, if perceived to be long-lasting, that would in fact raise debt ratios.
Nonetheless, a rate cut is widely expected next month.

Sweden’s experience is a cautionary tale for other central banks. In Britain, house
prices have risen by 10% in the past year, and household debt, though down
relative to income since the recession, still exceeds 140% of income. Mark Car-
ney, the governor of the Bank of England, complains this has “unbalanced” Brit-
ain’s recovery. He has hinted at countermeasures such as higher capital charges
for mortgages and limits on high loan-to-value or debt-to-income ratios. He also
warned that interest rates could rise sooner than markets expect, although this
was motivated more by the surprising pace of recovery than by fears of financial
instability.

Ms Yellen’s challenge is different. America’s economy, in contrast to Britain’s,


continues to disappoint: on June 18th policymakers lowered their projection for
economic growth this year, to 2.2% from 2.9% in March, the latest in a series of
downgrades. They also reaffirmed their expectation that rates would stay near
zero through the middle of next year, though the projected pace of tightening
thereafter rose slightly.

Two things could prompt an earlier move. One is prices. The trend of falling infla-
tion that had alarmed Fed officials is now over: consumer prices were up by 2.1%
in May, the fastest in 20 months. But that is hardly unnerving; according to the
Fed’s preferred index inflation is closer to 1.6%, well below the 2% target, and it
is expected to stay below target through 2016. Moreover, Fed officials have re-
cently hinted that they would prefer to let inflation rise above 2% briefly than stran-
gle the recovery with premature tightening.

Before the meeting the IMF, in an unusually public and explicit fashion, urged the
Fed to keep rates near zero for even longer than now planned and to let inflation
exceed its target. But the dilemma this creates is well illustrated by the IMF’s
indecision on the issue. In April it cautioned that “undue delay” in raising interest
rates “could lead to a further build-up of financial stability risks”.

American homes are reasonably valued and household debt is just 109% of in-
come, and falling. But fixed-income markets exhibit euphoria reminiscent of 2007.
A record $94 billion of junk bonds was issued in the second quarter, and lever-
aged loans are on track to match last year’s record $1.1 trillion, according to
Thomson Reuters. A third of those loans lack the standard covenants that limit
how much debt the borrower can carry.

11 de 84
EXAMEN DE ACTUALIZACION Y SELECCIÓN BCRP
COMPILADOR MARINO ENEQUE

Ms Yellen said such trends are not troubling enough to affect the course of mon-
etary policy for now; she prefers that they be dealt with through “macroprudential”
policy, meaning regulations designed to address specific imbalances in the finan-
cial system. The Fed has already urged tougher underwriting standards on lev-
eraged loans. But some of her colleagues want higher interest rates to play a part
as well. Esther George, president of the Federal Reserve Bank of Kansas City,
recently warned that the Fed’s commitment to keep interest rates low has encour-
aged small banks to raise the share of their bondholdings and loans that mature
in more than three years to 53%, up from 37% in 2005. That exposes them to
“heavy losses” if rates rise suddenly. Jeremy Stein, a Fed governor until last
month, has noted that moneymen can find ways around macroprudential con-
trols, whereas tighter monetary policy “gets in all of the cracks”.

Most officials at the Fed, however, worry more about an aborted recovery than
financial instability. Three Fed economists helpfully quantified this trade-off last
year by simulating two responses to a financial crisis. In one, rates fall to zero,
and another crisis, 60% as bad as the first, erupts ten years later. In the other,
rates are not allowed to fall below 1.5%, and the second crisis is avoided. Their
conclusion: even with the second crisis, the cumulative loss of income and em-
ployment is far smaller in the first scenario. The ironic and unsettling upshot is
that, sometimes, chancing another crisis is the optimal policy.

36. According to the reading, what was 37. Why the article mentions that the
the consensus before the global fi- Feds Challenges are different to
nancial crisis regarding the re- those of Britain.
sponse of central banks to asset
bubbles: a) Because, USA is facing a fast recov-
ery with inflationary pressures
a) Asset prices should be part of the re- b) Because, inflation in USA is picking
action of the central banks up and also asset bubble risks
b) Central banks should respond only c) Because economy growth recovery
after a bubble burst in USA is slow and inflation is sub-
c) Central banks should alert the mar- dued
ket about asset bubbles d) Because Britain economic growth is
d) Asset bubbles are a rational out- one of the fastest in the world
come of sophisticated financial mar- e) None of the above.
kets; therefore central banks should
not intervene 38. Why the article states that Riks-
e) None of the above bank’s stance has been deeply divi-
sive?

12 de 84
EXAMEN DE ACTUALIZACION Y SELECCIÓN BCRP
COMPILADOR MARINO ENEQUE

a) Because board members do not d) To fix the interest rates in the finan-
agree regarding liquidity policy cial system
b) Because some members of the e) None of the above
board believe monetary policy is too
tight 42. Reserve requirements are:
c) Because, board members believe a) A fraction of the deposits that banks
more macroprudential policies maintain at the central bank
should be put in place b) A special investment instrument for
d) Because board members do not banks
agree on the policy instrument to be c) A liability for banks
employed to deal with the recent
d) All of the above alternatives
slowdown of their economy
e) None of the above.
e) None of the above
43. The production function of a typical
39. What is macroprudential policy?
firm represents.
a) It is a set of policies oriented to fight
inflation a) The maximum amount of output that
the firm can produce for a given level
b) It is a set of policies oriented to pro-
of inputs
mote financial stability
b) The firm’s income
c) It is a set of policies that target ex-
change rate risk as its main goal c) The amount of dividends that the firm
can distribute
d) It is a type of monetary policy frame-
work d) B & C
e) None of the above e) None of the above

40. According to the reading, what is 44. The real exchange rate is.
the main concern of the officials at a) A relative price
the Fed? b) The value of the total production of
the economy
a) Financial stability
c) The price of domestic currency in
b) Global inflation
terms of a foreign currency
c) Slowdown in China
d) All of the above
d) An aborted recovery of the USA
e) None of the above
economy
e) None of the above. 45. Balance growth path means.
41. The main objective of the Peruvian a) The highest GDP growth rate possi-
Central bank is: ble
b) The sustainable long-term GDP
a) To promote price stability growth rate
b) To promote high GDP growth rates c) The short-term GDP growth rate
c) To prevent balance of payment crisis d) The per capital income
e) None of the above

13 de 84
EXAMEN DE ACTUALIZACION Y SELECCIÓN 2013

MACROECONOMÍA 2013

1.Considere el modelo de crecimiento a) Las economías que ahorran más


AK. Es posible asegurar en este mo- tienen un mayor nivel de PBI de
delo que: capital per cápita en estado estacio-
nario
a) La productividad marginal de- b) No hay crecimiento de largo plazo si
creciente del capital asegura no hay crecimiento de la productivi-
que no haya convergencia dad
b) La tasa de crecimiento está relacio- c) La convergencia es global
nada con la renta de los países. d) Para niveles de capital por encima
c) No hay transición hacia el estado es- de la regla de oro se produce una in-
tacionario eficiencia dinámica.
d) La tasa de crecimiento de las prin- e) Ninguna de las anteriores
cipales variables decrece con-
forme se acerca al estado estacio- 4.Considere un modelo de ciclos eco-
nario nómicos reales (RBC) estándar,
e) Ninguna de las anteriores ¿cuál de las siguientes informaciones
es correcta sobre el empleo?:
2.¿Cuál de las siguientes afirmaciones
es incorrecta sobre el modelo de cre- a) El desempleo originado en la econo-
cimiento neoclásico de Ramsey? mía es voluntario
b) La sustitución intertemporal del tra-
a) A diferencia del modelo de Solow, bajo no representa un mecanismo de
permite endogenizar la tasa de aho- propagación pues a través de ella los
rro shocks no se difunden ni se amplifi-
b) Cuando se introduce crecimiento can
tecnológico, el consumo per cápita c) Sin importar la función de utilidad
de la regla de oro coincide con el que se tenga, la respuesta del nú-
consumo per cápita de la regla de mero de horas trabajadas ante los
oro modificada choques de productividad siempre
c) Un aumento en la tasa de des- será positiva
cuento subjetiva del consumo sólo d) El salario real puede ser procíclico o
afecta a la senda de crecimiento contracíclico, dependiendo de la
del consumo, pero no a la senda fase del ciclo en la que nos encontre-
de crecimiento del capital mos
d) Cuando el modelo se resuelve en e) Ninguna de las anteriores
tiempo finito, la trayectoria óptima
bajo tiempo infinito deja de ser óp- 5.¿Cuál de las siguientes afirmaciones
tima en las últimas etapas del mo- no es correcta sobre la Ley de Okun?
delo a) Relaciona de manera inversa el pro-
e) Ninguna de las anteriores ducto y el desempleo
3.Según el modelo de Solow, ¿qué afir- b) En la formulación matemática se
mación es incorrecta? debe considerar el producto poten-
cial y la tasa natural de desempleo
EXAMEN DE ACTUALIZACION Y SELECCIÓN BCRP
COMPILADOR MARINO ENEQUE

c) Arthur Okun señaló que en promedio incentivos a generar una inflación


era necesario crecer 2% por cada alta.
1% de desempleo que se quería re- c) Adoptar un régimen de metas explí-
ducir citas de inflación
d) Existe una tasa mínima de creci- d) Adoptar un sistema de reglas que
miento para que el desempleo no se defina claramente los objetivos del
incremente Banco Central
e) Ninguna de las anteriores e) Ninguna de las anteriores
6.Los bancos centrales enfrentan un di- 9.Suponga que un país tiene una de-
lema de política entre estabilizar el manda de dinero muy estable y los
producto y estabilizar la inflación, shocks reales tienen una mayor va-
cuando la economía es afectada por: rianza que los shocks nominales. A
a) Shocks de productividad fin de reducir la volatilidad del pro-
b) Shock de costos ducto sería conveniente elegir como
c) Shocks de tasa de interés instrumento de política monetaria a:
d) Shocks de demanda a) La tasa de interés
e) Ninguna de las anteriores b) La oferta monetaria
7.De acuerdo con el canal crediticio, c) El tipo de cambio
una política monetaria expansiva: d) La inflación
e) Ninguna de las anteriores
a) Eleva el valor de los precios de los
activos financieros 10. En una economía con expectativas
b) Incentiva la inversión vía la “q” de forward looking, el anuncio de una
Tobin política monetaria contractiva puede
c) Incentiva una mayor demanda de ser expansiva si es que el anuncio al
exportaciones público:
d) Reduce el acceso a fondos presta- a) Reduce las expectativas de inflación
bles de la banca y reduce el crédito b) Reduce la demanda agregada de-
e) Ninguna de las anteriores bido a una mayor confianza empre-
8.Para reducir el problema del sesgo in- sarial
flacionario Keneth Rogoff: c) Desplaza ascendentemente la
Curva de Phillips
a) Contratar a un banquero central con d) Esta política no es anticipada por los
una aversión a la inflación mayor agentes económicos
que a la población. e) Ninguna de las anteriores
b) Incentivar la transparencia de la po-
lítica monetaria a fin de reducir los

15 de 84
EXAMEN DE ACTUALIZACION Y SELECCIÓN BCRP
COMPILADOR MARINO ENEQUE

11. El siguiente es un modelo Mundell-Fleming ampliado


Mercado de bienes:

𝑦𝑟 𝑑 = 𝛼1 (𝑒 − 𝑝𝑑 ) + 𝛼3 𝑔𝑟 − 𝛼4 𝑟 𝑑 … (demanda agregada)
Mercado de dinero:

𝑚𝑜𝑑 = 𝑝𝑑 + 𝛼5 𝑦𝑟 − 𝛼10 𝑟 𝑑 … (demanda por dinero)


𝑚𝑜 𝑠 = 𝑚𝑜 … (oferta de dinero)

Balanza de pagos:

𝐵 = 𝛼13 (𝑒 − 𝑝𝑑 ) − 𝑦𝑟 + 𝛼14 𝑟 𝑑
Mercado laboral:

𝑦𝑟 𝑠 = 𝛼9 (𝑤 − 𝑝𝑑 )
𝑤=𝑝 …. (indexación salarial perfecta)

𝑝 = 𝛼15 𝑝𝑑 + (1 − 𝛼15 )𝑒 …. (índice de precios)

𝑦𝑟 𝑠 = −𝛼9 (1 − 𝛼15 )(e − 𝑝𝑑 ) … (oferta agregada)

Sobre la base de este modelo, con tipo tivo, positivo. Sin esterilización: ne-
de cambio fijo, ¿qué efecto tendrá so- gativo, positivo, positivo.
bre 𝑦r, pd y 𝑟𝑑 una política fiscal expan- e) Ninguna de las anteriores
siva asumiendo esterizalización per- 12. Suponga un modelo IS-LM básico.
fecta y qué efecto cuando no exista es- Ante un aumento exógeno del gasto
terilización? fiscal, determine la veracidad o false-
a) Con esterilización: positivo, nega- dad de las siguientes afirmaciones
tivo, positivo. Sin esterilización: posi- i. Si la autoridad monetaria mantiene
tivo, positivo, negativo. constante el tipo de interés, aumen-
b) Con esterilización: negativo, posi- tará el consumo privado, mantenién-
tivo, positivo. Sin esterilización: posi-
dose constantes los niveles de inver-
tivo, positivo, negativo.
sión e ingreso.
c) Con esterilización: positivo, positivo,
ii. Si la elasticidad de la demanda por
positivo. Sin esterilización: positivo,
dinero respeto a tipo de interés es in-
positivo, positivo.
finita, la inversión y el consumo pri-
d) Con esterilización: negativo, posi-
vado no se verán afectados.
iii. Si la elasticidad de la demanda por

16 de 84
EXAMEN DE ACTUALIZACION Y SELECCIÓN BCRP
COMPILADOR MARINO ENEQUE

dinero respecto al tipo de interés es inversión privados. Estos argumen-


nula, la inversión privada disminuirá tos son:
y permanecerá constante el nivel de a) FFVF
consumo privado. b) FFVV
c) VFVF
iv. Si la autoridad monetaria mantiene d) FVVF
constante la cantidad de dinero, des- e) Ninguna de las anteriores
cenderán los niveles de consumo e

13. Dado el siguiente modelo Mundell - Fleming:


𝑌 = 𝐶 (𝑌) + 𝐼(𝑟) + 𝐺 + 𝑋 (𝐸, 𝑌 ∗ ) − 𝐸𝑀(𝑌, 𝐸)
𝑀𝑆 = 𝐷 + 𝑅
𝑀𝐷 = 𝑃𝐿(𝑌. 𝑟)
𝑀 𝑆 = 𝑀𝐷
𝐸̇
𝑅̇ = [𝑋(𝐸, 𝑌 ∗ ) − 𝐸𝑀(𝑌, 𝐸)] + 𝐾 [𝑟 − 𝑟̇ − ]
𝐸
¿Cuáles son las condiciones de estabilidad del modelo?

a) (1  CY  EMY )  Lr  0 ; Ir EMY  K r ´ (1  CY  EMY )  0 y

 (1  CY  EMY ) Ir   Lr 1  (1  CY  EMY ) 0 


   0
 LY Lr  K r ´ 0   EMY 0 

b) (1  CY  EMY )  Lr  0 ; Ir EMY  K r ´ (1  CY  EMY )  0 y

 (1  CY  EMY ) Ir   Lr 1  (1  CY  EMY ) 0 


   0
 LY Lr  K r ´ 0   EMY 0 

c) (1  CY  EMY )  Lr  0 ; Ir EMY  K r ´ (1  CY  EMY )  0


d) el modelo siempre es estable
e) Ninguna de las anteriores

14. De las siguientes afirmaciones: ii. Un aumento de los términos de inter-


cambio genera un aumento de la de-
i. El efecto Harrod-Balassa - Samuel-
son señala que un aumento de la manda de trabajadores en el sector
productividad del sector transable exportable que eleva los salarios en
sobre el no transable lleva a una el país local y consecuentemente
apreciación del TCR de equilibrio.
eleva el precio de los bienes no

17 de 84
EXAMEN DE ACTUALIZACION Y SELECCIÓN BCRP
COMPILADOR MARINO ENEQUE

transables y aprecia con ello el TCR d) El retorno de dos activos con igual
iii. Una mayor apertura comercial riesgo debe ser el mismo, si los re-
eleva la demanda de bienes impor- tornos se expresan en la misma mo-
tados y reduce relativamente la de-
neda
manda de bienes no transables lo
que deprecia el TCR e) Ninguna de las anteriores
iv. Menores aranceles requeriría un 17. La teoría de la paridad no cubierta
TCR más bajo para mantener la de tasas de interés se deriva de:
competitividad frente al exterior a) La condición de arbitraje entre los re-
¿Cuántas son verdaderas? tornos de activos con igual riesgo
pero expresados en distinta mo-
a) Una
neda.
b) Cuatro
b) La demanda de dinero relativa de
c) Dos
dos países.
d) Tres
c) La regla de Taylor
e) Ninguna de las anteriores d) a y b son ciertas
15. ¿Un shock positivo de términos de e) Ninguna de las anteriores
intercambio qué efecto tiene sobre la
cuenta corriente de una economía 18. El tipo de cambio forward se deter-
pequeña y abierta? mina a partir de :
a) Aumenta superávit o reduce déficit i) El diferencial de tasas de interés en
b) No tiene efecto moneda doméstica y externa.
c) Depende si el shock es permanente ii) El nivel de cuenta corriente sosteni-
o transitorio ble.
d) Depende si los términos de inter- iii) La condición de arbitraje entre el
mercado monetario doméstico y ex-
cambio aumentan por incremento terno.
de precios de exportación o por dis- Son ciertas:
minución de precios de importación a) Sólo i.
e) Ninguna de las anteriores b) Sólo ii.
16. La paridad de tasas de interés im- c) Sólo iii.
plica que: d) Sólo i y iii.
e) Ninguna de las anteriores
a) El retorno de dos activos en países
diferentes pero con igual riesgo de 19. Las políticas monetarias no con-
vencionales hacen referencia a:
crédito debe ser el mismo
b) La moneda doméstica en el país A a) El uso de instrumentos de política
se aprecia si la tasa de interés en el monetaria diferentes a la tasa de in-
país B sube terés
c) La moneda doméstica en el país A b) Una meta de inflación más alta
se deprecia si la tasa de interés en el c) El uso de la tasa de interés para es-
país B baja tabilizar el tipo de cambio

18 de 84
EXAMEN DE ACTUALIZACION Y SELECCIÓN BCRP
COMPILADOR MARINO ENEQUE

d) a y b son correctas responden más a cambios en las ta-


e) Ninguna de las anteriores sas de interés de corto plazo cuando
20. De los siguientes postulados: estás últimas tasas muestran cam-
i) Los déficits de cuenta corriente se bios persistentes.
generan por que el banco central Son falsas:|
acumula reservas internacionales. a) i) y ii)
ii) Mayores requerimientos de encajes b) ii) y iii)
permiten que los bancos expandan c) Sólo i)
el crédito al sector privado. d) Sólo iii)
iii) Las tasas de interés de largo plazo, e) Ninguna de las anteriores

MICROECONOMÍA 2013

21. Elmer tiene una función de utilidad del ingreso.


de la forma 𝑈 = con una res- a) 𝐼(1 − √2), 𝐼√2
𝑥 0.5 𝑦 0.5
tricción presupuestaria 𝑃1x +𝑃2𝑦 = 𝐼. b) 𝐼(1⁄√2 − 1), 𝐼 √2⁄2
Hallar la demanda del bien x.
c) 𝐼√2, 𝐼√2
a) 2𝑝
1
1 d) 𝐼(1⁄√2 − 1), 𝐼(1 − √2)
b) 2𝑝
𝐼
1 𝑝2
e) Ninguna de las anteriores
c)
2𝐼
24. En una economía llamada Narvnia
𝑝1
un hogar típico tiene un ingreso de
d)
𝐼
2𝑝12 168 soles y consume dos bienes: ali-
mentación (x) y vivienda (y). Las pre-
e) Ninguna de las anteriores ferencias del hogar típico son repre-
22. Ahora asumamos que el precio del sentadas por una función de utilidad
bien x se duplica. Hallar el cambio en del tipo Stone–Geary
el excedente del consumidor en fun- 𝑈(𝑥, 𝑦) = (𝑥 − 2)0.2 (𝑦 − 1)0.8 ,
ción del ingreso
donde 2 y 1 representan los consu-
a) 2 𝑙𝑛(2)
𝐼
mos mínimos de alimentación y vi-
vienda, respectivamente. Hallar el
b) 2
𝐼
mayor nivel de bienestar que un ho-
c) 𝐼 𝑙𝑛(2) gar típico puede alcanzar.
d) 2 𝑒
𝐼 2
a) 2,5
b) 3
e) Ninguna de las anteriores c) 3,5
23. En el contexto anterior, calcular la d) 4
variación equivalente y la variación e) Ninguna de las anteriores
compensatoria en función en función 25. Suponga que la función de utilidad
indirecta es:

19 de 84
EXAMEN DE ACTUALIZACION Y SELECCIÓN BCRP
COMPILADOR MARINO ENEQUE

𝐼 𝑝𝑦 noce que la función de utilidad espe-


+ 𝑉=
𝑝𝑦 4𝑝𝑥 rada del individuo está dada por
Hallar la demanda ordinaria para el bien 𝑈(𝑥) = 𝑥 0.5 , indique cuál es el valor
x. de probabilidad de robo que hace in-
diferente al consumidor entre com-
prar o no un seguro de casa
a) 1
4  Py 2
Px  a) 2%
b) 60%

 
2 Py 2 c) 20%
b) 1
8 Px d) 50%
e) Ninguna de las anteriores

c) 1
4  Py 2
2 Px  28. En la teoría de la utilidad esperada
de Von Neumann-Morgenstern se
cumple que:
 
Py 1/2
d) 1
2 Px
a) La noción de utilidad tiene un carác-
ter cardinal pero no ordinal
e) Ninguna de las anteriores b) La paradoja de Allais constituye una
prueba empírica de la validez de
26. Sean dos consumidores, 1 y 2, con esta teoría
A(w) y B(w) como funciones de utili-
c) Según esta, el valor de la utilidad es-
dad esperada de la riqueza, respec-
perada es mayor a la suma de las uti-
tivamente. Asuma que estas son di-
lidades de los posibles resultados,
ferenciables, crecientes y cóncavas.
ponderados por sus probabilidades
Indique cuál de las siguientes propo-
de ocurrencia.
siciones implican que el agente 1 es
d) Este tipo de función de utilidad no
más adverso al riesgo
permite obtener conclusiones sobre
a) El agente 1 está dispuesto a pagar la aversión relativa al riesgo.
más por una prima por seguro que e) Ninguna de las anteriores
el agente 2, dado un mismo nivel de
riesgo 29. La paradoja de Allais señala que:
𝐴‼ (𝑤) 𝐵‼ (𝑤) a) Agentes adversos al riesgo pueden
b) Se cumple: − ! (𝑤) > − !(𝑤)
𝐴 𝐵 aceptar loterías riesgosas
c) Si G es una función creciente y es- b) Agentes neutrales al riesgo siempre
trictamente cóncava, se cumple aceptan juegos justos (“fair games”)
que: 𝐴(𝑤) = 𝐺(𝐵(𝑤)) c) Agentes pueden ser irracionales e
d) Sólo a y b inconsistentes
e) Ninguna de las anteriores d) Existe inconsistencia entre las elec-
27. Suponga que un individuo puede ciones observadas empíricamente
elegir entre comprar un seguro de respecto a las predicciones de la teo-
casa contra robos por un valor de ría de la utilidad esperada
$19, y no hacerlo. Si es víctima de un e) Ninguna de las anteriores
robo podría perder hasta $36 del va- 30. Sea que un consumidor tiene una
lor total de sus bienes (los cuales es- riqueza inicial de $200, y existe la po-
tán valorizados en $100). Si se co- sibilidad de que pierda $50. Puede

20 de 84
EXAMEN DE ACTUALIZACION Y SELECCIÓN BCRP
COMPILADOR MARINO ENEQUE

suscribir un seguro por el cual reci- precio.


birá x dólares si experimenta dicha d) Si se reduce la producción aumentan
pérdida. La cantidad que debe pa- sus márgenes
gar por una cobertura de x dólares e) Ninguna de las anteriores
del seguro es de ∝x dólares; donde 33. Si el coeficiente de aversión al
∝ es la prima por dólar de cobertura. riesgo de un agente es alto, esto im-
Si se asume que el mercado de se- plica que:
guros es uno perfectamente compe- a) Le gusta poco suavizar consumo.
titivo (empresas generan beneficios b) Le gusta mucho suavizar consumo.
nulos), señale por qué monto se c) Su consumo responde mucho a
asegurará el individuo. cambios en la tasa de interés real.
a) $25 d) Su consumo aumenta cuando au-
b) Por más de $50 si es adverso al menta el precio del bien
riesgo, y menos de $50 si es amante e) Ninguna de las anteriores
al riesgo.
c) $50 34. La existencia de costos fijos en la
d) Solo contratará un seguro si es ad- producción implica que:
verso al riesgo a) El precio no se iguala al costo mar-
e) Ninguna de las anteriores ginal de la empresa
b) La empresa hace siempre pérdidas
31. Para el caso de dos bienes que
c) Existe un tamaño mínimo de mer-
son perfectamente sustitutos y se
cado para que opere la empresa
producen por dos empresas diferen-
d) La empresa cobra un precio por en-
tes es cierto que:
cima del costo marginal
a) Los costos totales de ambas empre- e) Ninguna de las anteriores
sas son los mismos
b) Los costos marginales de ambas 35. Cuando las preferencias de los
empresas son los mismos agentes no son homotéticas, esto im-
c) Los márgenes son los mismos. plica que:
d) Todas las anteriores son verdaderas a) El consumo de todos los bienes se
e) Ninguna de las anteriores duplica si el ingreso se duplica
32. Para un monopolista que produce b) El consumo de cada bien es cre-
un bien x, y tiene retornos margi- ciente en el nivel de ingreso
nales constantes, es cierto que: c) La canasta de consumo cambia con
a) Si aumenta la producción se reducen el nivel de ingreso
sus márgenes d) La canasta de consumo no cambia
b) Si aumenta la producción se aumen- con el nivel de ingreso
tan sus costos. e) Ninguna de las anteriores
c) Si aumenta la producción aumenta el

21 de 84
EXAMEN DE ACTUALIZACION Y SELECCIÓN BCRP
COMPILADOR MARINO ENEQUE

QUESTIONS- ENGLISH

Free exchange: A trio of trilemmas, The gold standard holds worrying les-
sons for the single currency (taken from The Economist, 06/07/2013)

EUROPEAN leaders have enjoyed a period of respite from financial turmoil since
last summer. But the euro remains vulnerable. Portuguese bond yields soared
this week as the ruling coalition fractured. Ireland’s economy has contracted for
three quarters in a row. A proper banking union is a long way off. The euro’s
fragility is underlined by a new study (http://www.nber.org/papers/w19112.pdf)
by Michael Bordo of Rutgers University and Harold James of Princeton
University. The two economic historians look at the flaws in another suppos-
edly impregnable international monetary regime, the gold standard, and find
reasons to fret about the single currency.
The parallels between the euro and the gold standard are not exact. The single
currency is a monetary union with the European Central Bank (ECB) at its apex;
the gold standard had no such institution. The euro floats against other currencies
such as the dollar, and the ECB is obliged to maintain price stability rather than
convertibility into gold. But for the 17 states that now share the single currency, it
represents a new gold standard in that their exchange rates with each other are
fixed.
That observation is not new, but the authors’ analysis of the tensions that eventu-
ally scuppered the gold standard is fresh. Those tensions, they argue, emerged
from a trio of “trilemmas”, each a set of three choices of which any two options
together are feasible but not all three. One of the three trilemmas is familiar to
economists: the “impossible trinity” of fixed rates, free movement of capital and an
independent monetary policy. This means that when currencies are locked and
capital can flow freely, countries surrender their ability to conduct their own mon-
etary policy. But Messrs Bordo and James reckon that countries in regimes like
the euro and the gold standard may not just be sacrificing their ability to set interest
rates; they may also be forsaking financial stability and even undermining democ-
racy.
Start with monetary independence. By throwing away the key of exchange rates,
countries must alter their relative domestic prices and wages when they become
misaligned. In its heyday before the first world war the gold standard worked well.
It generated pressures on both surplus and deficit countries when they respec-
tively gained or lost competitiveness. States with surpluses acquired gold, pushing
up the money supply, raising prices and making them less competitive. States with
deficits lost gold, which caused the money supply to shrink, pushing prices down
and sharpening their edge against rivals.
Unfortunately the euro resembles the flawed interwar version of the gold
standard rather than the classical pre-war model. After the gold standard was
restored in the 1920s, central banks in surplus states like France (which had
rejoined it at an undervalued exchange rate) sterilised the monetary effects
of gold inflows so that prices did not rise. That put all the pressure to adjust

22 de 84
EXAMEN DE ACTUALIZACION Y SELECCIÓN BCRP
COMPILADOR MARINO ENEQUE

on countries like Britain, which rejoined the gold standard in 1925 at an overval-
ued rate. A similarly harsh deflationary process is now under way in peripheral
euro-zone countries like Greece. Their adjustment would be much less draconian
if the core states were prepared to tolerate considerably higher inflation than the
euro-zone average. But Germany fiercely resists this.
The second of the authors’ trilemmas is the incompatibility of fixed exchange rates
and capital mobility with financial stability. When countries joined the gold stand-
ard, it bestowed a seal of approval that prompted a big influx of foreign money.
That pumped up credit, driving an expansion of domestic banks that often ended
in grief. Under the gold standard a strong state could support wobbly banks and
investors; in pre-war Russia, for example, the central bank was called the “Red
Cross of the bourse”. But a weak state could easily forfeit investors’ confidence,
as happened to Argentina in its 1890 debt-and-banking crisis. That same story has
been repeated in the brief history of the euro. Money cascaded into peripheral
Europe, causing banking booms and housing bubbles. In the bust that followed,
the task of recapitalising banks has caused both the Irish and Spanish states to
buckle.
The third trilemma is the most worrying: the potential incompatibility of fixed ex-
change rates and free movement of capital with democracy. Germany was able to
rejoin the gold standard after the first world war thanks to the confidence-boosting
Dawes Plan in 1924 dealing with reparation payments. But the harsh fiscal medi-
cine administered during the Depression in its effort to stay on gold contributed to
the rise of the Nazis. Britain left gold in 1931, presaging the end of the gold stand-
ard, because the austerity it required had become unbearable.

Demos and the demos


The potential for a similar backlash against the economic and fiscal require-
ments of Europe’s monetary union is clear. Although southern Europeans still
want to keep the euro, not least because the cost of exit is harsher than
that of leaving the gold standard, disenchantment grows. Italian voters said
basta to austerity in February; Portugal’s government is fraying in the face of
public hostility to tax rises and spending cuts. Northern Europeans are also
unhappy. Popular opposition to paying for euro- zone rescues constrains An-
gela Merkel, the German chancellor, from spelling out the sacrifices voters must
make to sustain the euro.
36. According to the reading, the third d) The lack of consensus to implement
dilemma is: regulatory reforms.
a) The lack of fiscal austerity e) None of the above
b) The potential lack of compatibility of 37. Why the article mentions that the
fixed exchange rates and free move- euro resembles the flawed interwar
version of the gold standard
ment of capital with democracy
a) Because, the ECB maintains high
c) The existence of the European Cen-
levels of reserves in gold.
tral Bank.

23 de 84
EXAMEN DE ACTUALIZACION Y SELECCIÓN BCRP
COMPILADOR MARINO ENEQUE

b) Because, it has a fixed exchange e) None of the above


rate among its members 41. Money supply is defined as:
c) Because the ECB is implementing
a) The liabilities of the Central Bank
non conventional monetary policy. b) The liabilities of the banks and the
d) Because the ECB has not cut its in- central bank
terest rate to zero c) The liabilities of the government
e) None of the above. d) a and b are true.
e) None of the above.
38. Why the article states that before
the First World War the gold stand- 42. GDP Deflator is used to:
ard worked well? a) Calculate Real GDP from Nominal
a) Because countries were able to ad- GDP
just their balance of payment misa- b) Measure the changes in Real GDP
lignments through changes in c) Inflation.
money supply. d) All of the above alternatives
b) Because there was more political e) None of the above
agreement during that period of 43. In consumption function c=a+by ,
time. the parameter a represents and y in-
c) Because, monetary policy was in-
come
dependent.
d) Because banks were better regu- a) The level of consumption when the
lated. income is zero.
e) None of the above. b) The slope of the function
c) MPC
39. What is a gold standard? d) b & C
a) It is a monetary system where e) None of the above
money is backed by gold.
b) Is the exchange rate used between 44. In open economy, where C is
gold and dollars before First World consumption, I, investment, G
War government expenditure, X, exports
c) It is a monetary system where cen- and M imports, GDP equals:
tral banks are independent. a) C+I+G+X-M
d) It is a monetary union b) C+I
e) None of the above. c) C+I+G
40. Why a single currency, like the d) All of the above
euro, faces dilemmas? e) None of the above
a) Because of lack of greater integra- 45. In the model Y=C+I+G and C=a+by
tion and political union Where b=0.6 , the expenditure multi-
b) Because of the increase their public plier is
spending a) 1
c) Because it is no possible to change b) 4
relative prices among its country c) 5
members d) 2,5
d) Because it is costly to implement e) None of the above
monetary policy

24 de 84
EXAMEN DE ACTUALIZACION Y SELECCIÓN BCRP
COMPILADOR MARINO ENEQUE

MACROECONOMÍA 2012
1. El modelo de oferta agregada de Lu- 1% de desempleo que se quería re-
cas desarrolla un tradeoff entre infla- ducir
ción y desempleo lo cual es posible al d) Existe una tasa mínima de creci-
asumir: miento para que el desempleo no
a) Información imperfecta suba
b) Precios rígidos e) Ninguna de las anteriores
c) Rigideces en los salarios nominales 4. Un aumento en el déficit de la
y mercados monopolísticos cuenta corriente no puede ser gene-
d) Mercados imperfectos rado por:
e) Ninguna de las anteriores a) Una disminución de las exportacio-
2. En una regla de Taylor el principio nes tradicionales
de Taylor asegura que: b) Aumento del ahorro externo
a) La respuesta del Banco Central en la c) Aumento en el consumo privado
tasa de interés nominal debe ser ma- d) Disminución en los ingresos del go-
yor a la inflación para afectar la tasa bierno por impuestos
de interés real. e) Un aumento en las transferencias
b) El Banco Central debe tener mayor corrientes
aversión por la inflación que por la 5. Señale la alternativa incorrecta so-
brecha producto bre los modelos de Ciclos Económi-
c) La regla de política monetaria (RPM) cos Reales (RBC):
debe ser derivada a partir de una re- a) La sustitución intertemporal del tra-
gla de Taylor bajo representa un mecanismo de
d) La respuesta del Banco Central propagación pues a través de ella
debe considerar las expectativas de los shocks se difunden y se amplifi-
inflación o inflación esperada can
e) Ninguna de las anteriores b) Los choques origina que la econo-
3. ¿Cuál de las siguientes afirmacio- mía se aleje del estado estacionario
nes no es correcta sobre la Ley de c) Los ciclos económicos representan
Okun? fluctuaciones de una economía que
a) Relaciona de manera inversa el pro- no está en equilibrio
ducto y el desempleo d) Las familias deciden sobre su oferta
b) En la formulación matemática se de trabajo y su senda de consumo
debe considerar el producto poten- e) Los pagos a los factores de produc-
cial y la tasa natural de desempleo ción dependen de sus productivida-
c) Arthur Okun señalo que en promedio des marginales
era necesario crecer 2% por cada 6. El progreso tecnológico a la Harrod
se caracteriza por:

25 de 84
EXAMEN DE ACTUALIZACION Y SELECCIÓN BCRP
COMPILADOR MARINO ENEQUE

a) La relación trabajo-producto se man- d) No se puede mantener una política


tiene constante monetaria independiente con un ré-
b) La productividad del capital aumenta gimen cambiario fijo y perfecta movi-
en la misma proporción que el aho- lidad de capitales
rro de capital y el trabajo e) Ninguna de las anteriores
c) La relación capital producto se man- 9. La Ley de Paridad de Poder de
tiene constante Compra relativa definiendo el tipo de
d) Ninguna de las anteriores cambio en términos de unidades mo-
7. Sobre el modelo IS-LM básico (eco- netarias domesticas por unidad mo-
nomía pequeña y cerrada) es cierto netaria extranjera, implica:
que: a) El tipo de cambio real es igual a 1
a) En el corto plazo el nivel del producto b) La tasa de variación del tipo de cam-
se determina en la IS en tanto que la bio es igual a la diferencia entre in-
tasa de interés en la LM flación externa e interna
b) En el corto plazo la política fiscal es c) La tasa de variación del tipo de cam-
totalmente inefectiva porque el con- bio es igual a la diferencia entre in-
sumo de las familias es desplazado flación interna y externa
por el consumo del gobierno d) El tipo de cambio de equilibrio es
c) En el corto plazo un aumento de la igual al ratio entre precios externos e
cantidad de dinero disminuye la tasa internos
de interés por el aumento de la de- e) (a) y (c) son verdaderas
manda de bonos 10. De los siguientes postulados:
d) Las opciones (a) y (c) son correctas. i) De acuerdo a las cuentas nacionales.
8. En el contexto del modelo Mundell- El Producto Nacional Bruto (PNB) es
Flemming, la trinidad imposible se igual al producto Interno Bruto (PIB)
refiere a: más el pago neto de factores
a) No se puede mantener una política ii)Una burbuja del precio de los activos
monetaria independiente con un ré- se refiere a que el aumento de los
gimen cambiario flexible e imper- precios de los activos se basa sola-
fecta movilidad de capitales mente en el aumento del retorno ac-
b) No se puede mantener una política tual (descontado) que los activos ge-
fiscal independiente con un régimen neran
cambiario fijo e imperfecta movilidad iii) Una regla fiscal estructural es pro-
de capitales cíclica ya que en periodos de auge la
c) No se puede mantener una política autoridad fiscal ahorra más, en tanto
monetaria independiente con un ré- que en épocas de contracción, ahorra
gimen cambiario flexible y perfecta menos. Son ciertas:
movilidad de capitales a) Solo i
b) Solo ii

26 de 84
EXAMEN DE ACTUALIZACION Y SELECCIÓN BCRP
COMPILADOR MARINO ENEQUE

c) Solo iii e) solo iii


d) Solo i y iii 13. La trinidad imposible significa
e) Solo i y ii que en una economía no es posible
11. Respecto al régimen de metas ex- tener el mismo tiempo:
plicitas de inflación el Banco Central: a) Tipo de cambio múltiple. Flujo libre
a) Fija el tiempo de cambio para cu- de capitales y política monetaria au-
brirse de su voluntad tónoma
b) Anuncia un rango objetivo para la in- b) Tipo de cambio flexible. Flujo libre de
flación para anclar las expectativas capitales y política monetaria autó-
de inflación noma
c) Propone una meta de corto plaza de c) Tipo de cambio fijo. Control de capi-
inflación, sin importar caer en incon- tales y política monetaria autónoma
sistencia inter temporal d) Tipo de cambio fijo. Flujo libre de ca-
d) Enfatiza como objetivo reducir el pitales y política monetaria autó-
desempleo noma
e) b y c son correctas e) Ninguna de las anteriores
12. De los siguientes postulados: 14. cuanto un país sufre del síndrome
i) La aplicación de políticas macropru- holandés
denciales debe tener como objetivo a) implica una salida masiva de divisas
evitar el riesgo sistémico en el sis- b) la moneda del país se fortalece
tema financiero y debe reemplazar a c) las exportaciones tradicionales del
la política monetaria país pierden competitividad bajo un
ii)La importancia de las RIN radica en esquema de tipo de cambio flexible
la capacidad que le otorgan a un país d) a y b son correctas
para cubrirse de choques adversos. e) b y c son correctas
Como por ejemplo una corrida de ca- 15. En el modelo de crecimiento de
pitales Ramsey:
iii) La mayor integración del sistema fi- a) La tasa de ahorro es endógena ya
nanciero conlleva a que exista un que se modela el consumo
análisis de red para poder conocer a b) El resultado basado en la regla de
que riesgos está expuesto el país en oro del ahorro no es un óptimo de
base a las interconexiones con distin- Pareto
tas entidades financieras
c) Sugiere que una política económica
Son falsas: destinada a incentivar el ahorro no
a) i y ii tiene impacto sobre el PBI per cápita
b) ii y iii d) El consumo y capital per cápita cre-
c) solo i cen en el largo plazo
d) solo ii e) Ninguna de las anteriores

27 de 84
EXAMEN DE ACTUALIZACION Y SELECCIÓN BCRP
COMPILADOR MARINO ENEQUE

16. Ante un incremento permanente b) Las tasas de interés aumentan de


de la cantidad de dinero, en una eco- manera transitoria
nomía con precios flexibles, debe c) El nivel de precios aumenta de ma-
cumplirse que: nera permanente
a) La inflación aumenta de manera per- d) La opción a y b son ambas verda-
manente dera
b) Las tasas de interés aumentan de e) Ninguna de las anteriores.
manera permanente. 19. Cuando la tasa de interés real está
c) El nivel de precios aumenta de ma- por encima de la tasa natural de inte-
nera permanente rés, todo lo demás constante.
d) La opción a y c son ambas verdade- a) La brecha producto es positiva.
ras b) La inflación está por debajo de la
e) Ninguna de las anteriores. meta del banco central.
17. Ante un incremento transitorio de c) La política monetaria es expansiva.
la cantidad de dinero, en una econo- d) La opción a y c son ambas verdade-
mía con precios flexibles debe cum- ras.
plirse que: e) Ninguna de las anteriores.
a) La inflación aumenta de manera 20. En un esquema de inflación es
transitoria cierto que el Banco Central:
b) Las tasas de interés aumentan de a) Siempre eleva la tasa de interés ante
manera permanente. desvíos de la inflación del rango
c) El nivel de precios aumenta de ma- meta.
nera permanente. b) Eleva la tasa de interés cuando hay
d) El nivel de precios se reduce de ma- choques de oferta.
nera transitoria. c) Eleva la tasa de interés cuando hay
e) Ninguna de las anteriores. choques de demanda.
18. Ante un incremento permanente d) Eleva la tasa de interés cuando los
de la tasa de crecimiento de la canti- choques afectan expectativas de in-
dad de dinero en una economía con flación.
precios flexibles, debe cumplirse que: e) La opción c y d son ambas verdade-
a) La inflación aumenta de manera per- ras.
manente

28 de 84
EXAMEN DE ACTUALIZACION Y SELECCIÓN BCRP
COMPILADOR MARINO ENEQUE

PREGUNTAS – MICROECONOMÍA 2012

21. Mercado de automóviles de se- 22. Pseudoseñalización (precio de


gunda mano (sin señalización). Los reserva “públicos”). De la pregunta 1,
vendedores de autos usados valoran suponga ahora que los compradores
sus coches (precio de reserva) como: conocen la forma funcional del precio
4vi de reserva. Determine el equilibrio
ri  , donde vi (de informa- para este caso
5
ción pública) corresponde a la valora- a) Se venden solo los coches de cali-
ción de los compradores por el coche dad 2
de calidad i. además lo siguiente: b) Se venden ambos tipos de coches a
Existen 2 tipos de coches, tal que: v = precios diferenciados
1
1000 y v2=200 c) Ningún tipo de coche se vende
Los compradores desconocen a d) Se venden los autos de calidad 1 a
priori si están comprando un auto de un precio de 1500
calidad 1 ó 2. e) Se vende los autos de calidad 1 a un
Los vendedores gozan de todo el precio de 1000
poder de negociación (precio = valo- 23. Señalización (multas). De la pre-
ración de los compradores). La forma gunta 1, asuma ahora que se ha es-
funcional de sus precios de reserva tablecido un mecanismo por el cual si
es información privada y no pueden un agente vende un auto de calidad
señalizar la calidad de sus coches. distinta a la ofrecida es sancionado
Asimismo considere que para un con una penalidad de 1100 a favor
comprador resulta igualmente proba- del comprador “estafado” (por solici-
ble adquirir cualquier de estos 2 ti- tud de éste). Describa como sería el
pos. Describa el equilibrio en este nuevo equilibrio.
mercado. a) Ambos tipos de coches se venden a
a) Se venden sólo los coches de cali- un mismo precio
dad 2. b) Ambos tipos de coches se venden a
b) Se venden ambos tipos de coches a precios diferenciados
un mismo precio. c) Los vendedores optan por no partici-
c) Ningún tipo de coche se vende. par
d) Se vende los autos de calidad 1 a un d) Los compradores optan por no parti-
precio de 1500. cipar
e) Se venden los autos de calidad 1 a e) c y d
un precio de 1000 24. Señalización (certificaciones). De
la pregunta 1, considere ahora que el

29 de 84
EXAMEN DE ACTUALIZACION Y SELECCIÓN BCRP
COMPILADOR MARINO ENEQUE

gobierno ha decretado que todos los d) Sólo se venden los coches de cali-
vendedores dispongan de un certifi- dad 2
cado de calidad para efectuar la e) Sólo se venden los coches de cali-
venta de sus automóviles. La agencia dad 1
encargada de otorgar estas certifica-
26. Un consumidor en una economía
ciones cobra 300 por cada emisión
de dos bienes tiene demandas de
(independiente de la calidad). Des-
tipo x(p,w) que satisfacen la ley de
criba cómo sería el equilibrio en este
Walras. Su función de demanda del
caso
primer bien es X1(p,w)= (1 – a) w/p1.
a) Solo se venden un tipo de coche ¿Cuál es la otra función de demanda,
b) Ambos tipos de coches se venden a es decir,X2(p,w) ? ¿Esta segunda
un mismo precio función es homogénea de grado
c) Ambos tipos de coches se venden a cero?
precios diferenciados (1  a) w
d) El precio de equilibrio es 1000 a) La función es X2 (p,w)= no
p2
e) b y d
es homogénea de grado cero.
25. Señalización (contratos y costes
de transacción). De la pregunta 1, p1
b) La función es X2 (p,w) = a w no
adicionalmente se establece que por p2
ley se debe estipular en los contratos es homogénea de grado cero.
de compra/venta “la cláusula de re-
venta”. La misma que consiste en p1
c) La función es X2 (p,w) = (1  a) w
otorgar la potestad al comprador de p2
devolver el auto recibiendo a cambio si es homogénea de grado cero
el 100 por ciento del precio pagado
tras un periodo de prueba (el necesa- w
d) La función es X2 (p,w) = a si es
rio para averiguar la calidad de co- p2
che). Por su parte, considere también homogénea de grado cero.
que existe una comisión por gastos
e) No se puede determinar
legales de 10 por cada “reventa” (dis-
tribuido en montos iguales entre las 27. Si una industria ignora la externali-
partes). Describa el equilibrio en este dades de costos que genera en su
escenario producción. ¿Cuál de las siguientes
alternativas es cierta bajo el supuesto
a) Ambos tipos de coches se venden a
de que existe producción en un mer-
precios diferenciados
cado competitivo?
b) Ambos tipos de coches se venden a
a) El precio será mayor que el costo
un mismo precio
marginal social
c) Ningún coche se vende

30 de 84
EXAMEN DE ACTUALIZACION Y SELECCIÓN BCRP
COMPILADOR MARINO ENEQUE

b) El precio será menor que el costo a) En el primer caso, hay un equilibrio.


marginal social (B, D). En el segundo caso. También
c) El precio será igual al costo marginal hay un solo, equilibrio que es (A,I)
social b) En el primer caso. Hay un equilibrio,
d) El costo marginal privado será igual (A, I). en el segundo caso, también
al costo marginal social hay un solo equilibrio que es (B.D)
e) El costo privado será mayor que el c) En el primer caso existen sólo equili-
costo marginal social brios en estrategias mixtas. En el se-
gundo, el único equilibrio es (A, D)
28. Una inversionista tiene como única
riqueza una acción de una empresa. d) En el primer caso existen solo equili-
Mañana esta acción puede valer S/. brios en estrategias mixtas. En el se-
36 o S/. 100. Ambos con la misma gundo, el único equilibrio es (B,I)
probabilidad (0.5 por ciento). El con e) En el primer caso, hay dos equili-
respecto al dinero (x) igual a u(x) = brios (A,I) y (B,D). en el segundo
caso, sólo hay un equilibrio que
u ( x)  x . ¿Cuál es el equiva- es(A, D)
lente cierto de la acción? ¿El inver- 30. En un duopolio de Stackelberg
sionista estaría dispuesto a vender la donde que la firma 1 es la líder, las
acción si alguien le ofreciese S/. 66 firmas conocen los costos y demanda
con seguridad?. de su rival. Las firmas enfrentan una
a) El equivalente cierto es S /.64; si es- demanda p = 100 – (q1 + q2) y los cos-
taría dispuesto a venderlo.
c q
2
tos son c1 = 100q1 y .
b) El equivalente cierto es S/. 64; no es- 2 2
taría dispuesto a venderlo. ¿Cuándo de más o de menos produ-
c) El equivalente cierto es S: 68: no es- ciría cada firma si cambiara de Sta-
taría dispuesto a venderlo. ckelberg a Cournot?. Redondee sus
d) El equivalente cierto es S/. 68; no es- resultados a 0 decimales.
taría dispuesto a venderlo. a) q1  0.8 q2  0
e) El equivalente cierto es S/. 68; pero
no puede determinarse si vende o no b) q1  0. q2  2
29. Considere el juego siguiente: c) q1  6. q2  2
I D
d) No se puede comparar ambos equi-
A 4:4 F:F librios
B 2:1 1:3 e) Ninguna de las anteriores
Si F = 1 ¿Cuál o cuáles son los equili- 31. La curva de beneficio marginal de
brios de Nash en estrategias puras?, y la economía para un bien público es
si F > 4, ¿Cuál es o son?: igual al:

31 de 84
EXAMEN DE ACTUALIZACION Y SELECCIÓN BCRP
COMPILADOR MARINO ENEQUE

a) Suma horizontal de las curvas de be- que establece su costo marginal. La


neficio marginal individuales grafica muestra la demanda de Agua
b) Suma horizontal de las curvas de de- de las Montañas. El costo marginal
manda individuales. es de $0.20 por botella. El precio de
c) Suma vertical de las curvas de bene- una botella de Agua de las Montañas
ficio marginal individuales. es ________ y se venden ________
mil botellas.
d) Suma vertical de las curvas de oferta
individuales.
80
e) Ninguna de las anteriores.

Precio (centavos por botella)


32. Un cartel obtendrá el mismo bene-
ficio económico que un monopolio
obtendría si:
a) El juego se juega una vez.
b) El cartel juega el juego una vez y hay D
una estrategia de devolver golpe por 0 800
golpe.
Cantidad (miles de botellas por mes)

c) El cartel gasta dinero en investiga- a) $0.50:300


ción y desarrollo. b) $0.50: 150.
d) El juego se repite y hay una estrate- c) $0.30: 600.
gia de devolver golpe por golpe. d) $0.20:300
e) Ninguna de las anteriores. e) Ninguna de las anteriores.
33. Suponga que se establece un im- 35. Una industria competitiva está en
puesto sobre las ventas de un bien, equilibrio de largo plazo. Algunas em-
pero el precio no cambia. Una situa- presas de la industria adoptan una
ción así ocurrirá si la__________ tecnología nueva que reduce el costo
bien es_______: total promedio de producción del
a) Demanda del; perfectamente elás- bien. En el largo plazo, el precio es
tica. _______, y las empresas con la tec-
b) Demanda del; de elasticidad uno. nología nueva obtienen beneficios
c) Oferta del; perfectamente elástica. _______ y las empresas con la tec-
nología antigua _______.
d) Demanda del; perfectamente inelás-
tica a) Mas bajo; normales; Se cambian a la
tecnología nueva o salen de la in-
e) Ninguna de las anteriores
dustria
34. Agua de las Montañas es un mo-
b) Más bajo; normales; salen de la in-
nopolio natural. El gobierno decide
dustria.
regular. Agua de las Montañas impo-
niendo una regulación a la forma en

32 de 84
EXAMEN DE ACTUALIZACION Y SELECCIÓN BCRP
COMPILADOR MARINO ENEQUE

c) Constante; económicos; obtiene be- d) Constante; normales; salen de la in-


neficios normales dustria
e) Ninguna de las anteriores

QUESTION ENGLISH 2012


The Euro Crisis: A delicate proposal (taken from The Economist, 26/06/2012)
HOW TO resolve the euro`s woes? Angela Merkel says Germany will not agree
to pool sovereign debt or share banking liabilities with other countries until there
is greater political union. Francois Hollande says France cannot accept the loss
of sovereignty without greater solidary. So today Herman Van Rompuy, the pres-
ident of the European Council (who chairs European summits), issued a report
that tries to Split the difference: there should be both joint liabilities and more
European-level control of national policies.
Drafted with three others – Jose Manuel Barroso, president of the European
Commission. Mario Draghi, president of the European Central Bank, and Jean-
Claude Juncker president of the Euro group (made up of finance ministers of the
euro zone) the report is thus a delicate piece of diplomacy at a time of Sharp
differences between Germany and France indeed between Germany and many
of the euro zone`s biggest countries.
It sets out four “building blocks” for deeper euro-zone integration a European sys-
tem to guarantee bank deposits and manage troubled Banks; fiscal integration to
exert greater control on national budgest and steps towards issuing joint debt,
more integration in economic policy-making to boost competitiveness; and more
democratic accountability in European-level decisions. Precisely what these
building blocks consist of how the steps should be sequenced and how much can
be done within existing treaties is all left a bit vague. The paper is peppered with
words like “appropriate” and “commensurate” without attempting to define them.
On what many in Brussels call “banking union”, it proposes creating a fund, “pri-
marily funded” through contributions from Banks, to help guarantee deposits and
wind up failed Banks. To be credible the fund should be backstopped by the euro
zone´s permanent rescue fund, the European Stability Mechanism (ESM), ex-
pected to be activated in the coming weeks.
On fiscal union, the report goes beyond the current system of euro-zone control
son national budget. It suggests jointly setting the ceiling for each country´s debt
and deficit. Governments would have to seek permission to break these limits
and Brussels could demand changes. Greater budgest controls could, later on,
lead to some form of Eurobonds (or “Eurobills” for short-date debt). This is how
the report puts the delicate issue.

33 de 84
EXAMEN DE ACTUALIZACION Y SELECCIÓN BCRP
COMPILADOR MARINO ENEQUE

In a medium term perspective, the issuance of common debt could be explored


as an element of such a fiscal union and subject to progress son fiscal integration.
Steps towards the introduction of joint and several sovereign liabilities could be
considered as long as a robust framework for budgetary discipline and competi-
tiveness is in place to avoid moral hazard and foster responsibility and compli-
ance. The process towards the issuance of common debt should be criteria-
based and phased, whereby progress in the pooling of decisions on budgets
would be accompanied with commensurate steps towards the pooling of risks .
Several options for partial common debt issuance have been proposed, such as
the pooling of some short-term funding instruments on a limited and conditional
basis, or the gradual roll-over into a redemption fund. Different forms of fiscal
solidarity could also be envisaged. Even further in the future, the euro zone could
create a common treasury with some kind of central budget.
The report has been toned down in drafting, and some contentious ideas have
been dropped. Gone is the call, made in an earlier version., for an “immediate
and permanent” mutualisation of risk to backstop the banking sector, Ditto for the
idea that the ESM could recapitalise Banks directly (so relieving some troubled
sovereigns like Spain from having to take on a big additional debt burden). The
report is also silent on the question of a financial-transactions tax, no doubt to
avoid upsetting a British government that is wary of stronger European bank su-
pervision.
It is worth comparing the wooliness of Mr. Van Rompuy´s paper with the detail of
the more convincingly worked-out proposal (here) issued on the same day by a
group of prominent economists under the aegis of Jacques Delors, the former
commission president and Helmut Schmidt, the former German chancellor. It
calls, for instance, for the creation of a special fund to help countries weather
cyclical downturns. It also proposes a European debt agency to oversee the is-
suance of limited amounts of joint Eurobonds (typically up to 10% of GDP, with
more possible if a country is willing to cede ever more budgetary decision-making
powers).
Mr. Van Rompuy´s report does not pretend to be a “roadmap” to greater fiscal
federalism. It is instead a proposal to talk about one, European officials argue
that just getting Mrs. Merkel to agree in principle to discuss thing like the mutual-
isation of debt would be a big achievement, as would getting the French to talk
about surrendering the powers of the Fifth Republic. Yet talking is one thing,
agreeing quite another-and there is still no sign of accord among leaders.
Mr. Van Rompuy says a detailed plan for integration could be presents in Decem-
ber, with perhaps an interim version issued in October. But will the markets be
ready to wait that long? After all, this is the week that Cyprus asked for a bail-out,
and Spain confirmed it would seek up to € 100 billion to recapitalise its Banks.

34 de 84
EXAMEN DE ACTUALIZACION Y SELECCIÓN BCRP
COMPILADOR MARINO ENEQUE

The likelihood is that this week´s European Summit will disappoint, and that could
set off another round of panic in the markets. If the euro is to survive, Mr. Van
Rompuy may have to draft his roadmap rather sooner than he expects.

36. According to the Reading the lead- d) Banks should avoid risks by invest-
ers of the European Union do not ing only in mutual funds
reach an agreement because: e) None of the above
a) Germany has the largest public debt
in the Euro zone 39. From the Reading, Angela Merkel
b) Germany is requiring that other is against.
country member adopt drastic fiscal a) Any Kind of financial support to trou-
adjustments bled European Countries
c) Greece is successfully implementing b) And Kind of monetary easing from
fiscal reforms the European Central Banks
d) Spain is the country that shows the c) Pool sovereign debt or share bank-
fasted recovery in the Euro-zone ing liabilities until there is greater po-
e) None of the above litical union
d) Buying bonds issued by Spain or
37. What is the main goal of the Bank- Greece
ing Union e) None of the above
a) To help European governments to fi-
nance public debts 40. From the Reading what would you
b) To reduce the need of liquidity of Eu- consider the most critical challenged
ropean Banks is for the European Union
c) To provide financial support to failed a) Greater integration and political un-
Banks ion
d) To buy government bonds b) Increase their public spending
e) To buy the new Eurobonds c) Cut investment in research
d) Tax more heavily banks
38. What the authors mean by “imme- e) None of the above
diate and permanent mutualisation of 41. The difference between real and
risk to backstop the banking sector” Nominal GDP is:
a) European countries should share a) measured by excluding of the sec-
the burden of all European countries tors
public debt b) that real GDP is always smaller
b) European funds, such us de EMS than Nominal GDP
should recapitalize directly banks c) Change in Price level from base
c) Banks should be transformed in mu- year to current year
tual funds d) None of the above

35 de 84
EXAMEN DE ACTUALIZACION Y SELECCIÓN BCRP
COMPILADOR MARINO ENEQUE

42. GDP Deflator is used to: 44. In closed economy GDP equals
a) Calculate Real GDP from Nominal a) C+I+X–N
GDP b) C+I
b) Measure the changes in Real GDP c) C+I+G
c) Inflation d) All of the above
d) Both of the above e) None of the above
e) None of the above
45. In the model Y= C + I + G and C =
43. In consumption function c = a + by, a – by Where b = 0.8. The expendi-
the parameter b represents ture multiplier is
a) The intercept a) 1
b) The slope of the function b) 4
c) MPC c) 5
d) b & C d) 8
e) None of the above e) None of the above

36 de 84
EXAMEN DE ACTUALIZACION Y SELECCIÓN BCRP
COMPILADOR MARINO ENEQUE

BCRP : MACROECONOMIA 2011


1. En un modelo intertemporal de dos c) Bajo expectativas racionales,
periodos estándar, si la tasa de des- cuando la inflación es más alta que
cuento subjetiva es mayor a la tasa lo esperado, el desempleo es alto.
de interés, entonces: d) a y b son correctas
a) Al individuo le conviene postergar e) a, b y c son correctas
consumo 4. En el modelo IS – LM (economía pe-
b) Al individuo le conviene adelantar queña y cerrada) ante un aumento
consumo. en el gasto fiscal, lo siguiente es
c) Para el individuo siempre es óptimo cierto sobre el nuevo nivel de equili-
consumir la misma cantidad en am- brio:
bos periodos. a) El aumento en la demanda agregada
d) La respuesta es ambigua es exactamente proporcional al mul-
2. Sobre el modelo de Solow (sin de- tiplicador fiscal
preciación ni progreso tecnológico) b) La demanda agregada aumenta en
¿Cuál de las siguientes afirmaciones menor proporción que lo predicho
es correcta? por el multiplicador fiscal.
a) La tasa de ahorro se determina de c) El nivel de saldos reales de dinero
manera endógena aumenta
b) Se asume retornos de escala cre- d) La tasa de interés se reduce
cientes e) Ninguna de las anteriores
c) En el estado estacionario, el pro- 5. En el modelo de Mundell – Fleming
ducto per cápita depende de la tasa con tipo de cambio flexible y econo-
de crecimiento de la población mía pequeña:
d) La regla de oro ilustra como un incre-
a) La política monetaria no puede te-
mento en la tasa de ahorro siempre
ner ningún efecto sobre la demanda
es asociado a mejoras de bienestar
agregada
3. Sobre la relación entre la inflación y b) El efecto de un aumento en el gasto
desempleo: fiscal es atenuado por una reducción
a) Bajo expectativas adaptivas, en el en las exportaciones netas
corto plazo puede haber dilema en- c) El efecto de un aumento en el gasto
tre inflación y desempleo fiscal es magnificado por un au-
b) Bajo expectativas racionales, no mento en las exportaciones netas
tiene que haber dilema entre la infla- d) a y c son correctas
ción y desempleo en el largo plazo
6. Sobre el modelo de Mundell – Fle-
ming:

37 de 84
EXAMEN DE ACTUALIZACION Y SELECCIÓN BCRP
COMPILADOR MARINO ENEQUE

a) La política fiscal tiene máxima efec- b) Cambios en la tasa de crecimiento


tividad sobre la demanda agregada del dinero en una economía afecta
bajo tipo de cambio flexible y nin- solo variables nominales
guna sobre tipo de cambio fijo c) Cambios en la cantidad de dinero en
b) La política monetaria tiene máxima una economía afecta solo variables
efectividad sobre la demanda agre- reales
gada bajo tipo de cambio fijo y nin- d) Cambios en la tasa de crecimiento
guna sobre tipo de cambio flexible del dinero en una economía afecta
c) a y b son correctas solo variables reales
d) Ninguna de las anteriores e) Ninguna de las anteriores
7. De los siguientes postulados: 9. La ley de Okun se refiere a:
i. Se entiende por capital humano al a) El costo de reposición de la inversión
stock de habilidades y experiencia es mayor que el gasto de inversión
que hace que los trabajadores sean b) Una mayor tasa de depreciación del
más productivos capital, menor será la producción
ii. La condición de paridad de tasas de c) Se espera una menor rentabilidad de
interés asume perfecta movilidad de la bolsa de valores, como resultado
capitales y establece que los retornos de una mayor incertidumbre
esperados entre diferentes países d) Existe una relación entre las fluctua-
son iguales en equilibrio ciones en desempleo y las fluctua-
iii. La tasa de referencia de la política ciones en el producto
monetaria sirve de señal para la de- e) Ninguna de las anteriores
terminación de la tasa de interés del
10.La curva de Phillips:
mercado interbancario
a) Relaciona positivamente el producto
Son verdaderas:
y la inflación
a) i y ii
b) Relaciona negativamente la inflación
b) ii y iii
y la tasa de empleo
c) solo iii
c) Relaciona negativamente el pro-
d) Todas ducto y la tasa de empleo
e) Ninguna d) a y b son verdaderas
8. La neutralidad del dinero implica e) Ninguna de las anteriores
que:
11.Con respecto a los encajes banca-
a) Cambios en la cantidad de dinero en rios:
una economía afecta solo variables
i. Una operación de mercado abierto
nominales
del Banco Central
ii. Se aplican a los depósitos en mo-
neda nacional

38 de 84
EXAMEN DE ACTUALIZACION Y SELECCIÓN BCRP
COMPILADOR MARINO ENEQUE

iii. Se aplican a los depósitos en mo- del retorno el siguiente año de un


neda extranjera bono a un año es:
a) i a) 10%
b) i y ii b) 20%
c) i y iii c) 2%
d) ii y iii d) 6%
e) Ninguna de las anteriores 14.En una economía pequeña y
12.De los siguientes postulados: abierta con perfecta movilidad de ca-
i. Una burbuja del precio de los activos pitales, tipo de cambio fijo y precios
se refiere a que el aumento de los rígidos:
precios de los activos se basa sola- a) Una expansión de la oferta moneta-
mente en la expectativa de que los ria genera una expansión del pro-
precios en el futuro serán altos. ducto
ii. Una burbuja del precio de los activos b) Una contracción en la demanda de
se refiere a que el aumento de los exportaciones genera una reducción
precios de los activos se basa sola- del producto
mente en el aumento del retorno ac- c) Un aumento en la tasa de interés in-
tual (descontado) que los activos ge- ternacional genera una ganancia de
neran. reservas internacionales netas (RIN)
iii. Regla de política monetaria se re- para el Banco Central
fiere a la relación sistemática entre d) a y b son correctas
como el banco central determina el e) Ninguna de las anteriores
uso de sus instrumentos de política
15.En una economía pequeña y
en reacción a movimientos en varia-
abierta con perfecta movilidad de ca-
bles como la inflación.
pitales:
Son falsas:
i) La política monetaria es útil sólo bajo
a) Solo i el régimen de tipo de cambio flexible
b) Solo ii ii) La política monetaria es útil sólo bajo
c) Solo iii un régimen de tipo de cambio fijo
d) Todas iii) La política fiscal es útil sólo bajo un
e) Ninguna régimen de tipo de cambio flexible
13.Si el logaritmo del retorno de un iv) La política fiscal es útil sólo bajo un
bono a dos años es 8% y el logaritmo régimen de tipo de cambio fijo
del retorno de un bono a un año es a) i) y iii) son correctas
de 14%, la hipótesis de expectativas b) i) y iv) son correctas
racionales de la tasa de interés im- c) ii) y iii) son correctas
plica que la esperanza del logaritmo
d) ii) y iv) son correctas

39 de 84
EXAMEN DE ACTUALIZACION Y SELECCIÓN BCRP
COMPILADOR MARINO ENEQUE

e) Ninguna de las anteriores iii) Un intento del Banco Central por re-
16.En una economía cerrada con pre- ducir la inflación ayudará también a
cios rígidos: reducir el desempleo natural.
a) Un aumento exógeno en la demanda Son verdaderos:
de dinero genera una expansión del a) i) y ii)
producto y un aumento en la tasa de b) ii) y iii)
interés. c) Sólo i)
b) Una contracción del gasto público d) Sólo iii)
aumenta el consumo privado.
e) Ninguno es verdadero
c) El Banco Central debe usar una re-
19.A propósito del Premio Nobel de
gla de política monetaria.
Economía 2010. En el modelo básico
d) El instrumento de política monetaria de búsqueda secuencial la principal
del Banco Central debe ser la tasa fuente de fricciones es:
de interés.
a) Fricciones por precios y/o salarios rí-
e) Ninguna de las anteriores. gidos.
17.La hipótesis del ingreso perma- b) Fricciones debido al poder de nego-
nente implica que: ciación de trabajadores
a) La propensión marginal a consumir c) Fricciones debido al poder monopó-
depende de si el aumento en el in- lico de trabajadores
greso es transitorio o permanente.
d) Fricciones de búsqueda de empleo
b) La propensión marginal a consumir
e) Ninguna de las anteriores
es alta para choques permanentes
al ingreso y baja para choques tran- 20.De acuerdo a la hipótesis de expec-
sitorios. tativas racionales
c) La propensión marginal a consumir a) Los agentes cambian sus expectati-
es baja para choques permanentes vas rápidamente cuando enfrentan
al ingreso y alta para choques tran- nueva información
sitorios. b) Los agentes usan más información
d) (a) y (b) son correctas que solamente datos pasados sobre
una variable para formar sus expec-
e) (a) y (c) son correctas
tativas sobre esta variable
18.De los siguientes postulados: c) Las expectativas sobre una variable
i) El producto potencial de la economía no difieren de las predicciones ópti-
depende del stock de dinero en la mas sobre éste usando toda la infor-
economía. mación disponible
ii) La tasa natural de desempleo de- d) Todas las anteriores
pende del nivel de inflación de largo
e) Solo (a) y (b) es verdad
plazo de la economía.

40 de 84
EXAMEN DE ACTUALIZACION Y SELECCIÓN BCRP
COMPILADOR MARINO ENEQUE

21.En un mundo con pocos impedi- 24.¿Cuál de los siguientes rangos de


mentos para la movilidad de capital, inflación está acorde con la meta de
la tasa de interés doméstica será inflación del BCRP?
igual a la suma de la tasa de interés a) De 0.0 % a 2.0 %
extranjera y la depreciación esperada b) De 1.0 % a 3.0 %
de la moneda local. Esta es una si-
c) De 0.5 % a 2.5 %
tuación que se conoce como:
d) No mayor a 2.0 %
a) Condición de paridad de poder de
compra e) No mayor a 3.0 %
b) Condición de paridad de tasas de in- 25.En el modelo Mundell – Fleming es-
terés tándar, en el corto plazo, un aumento
del tipo de cambio real genera.
c) Condición de paridad de tipo de
cambio a) Una reducción de la demanda agre-
gada y del producto
d) Condición de paridad externa de ac-
tivos b) Un aumento de la demanda agre-
gada y una reducción del producto
e) Ninguna de las anteriores
c) Un aumento de la demanda agre-
22.Una venta de bonos del gobierno
gada sin ningún cambio en el pro-
de parte de la autoridad monetaria
ducto
(banco central), ceteris paribus:
d) Un aumento de la demanda agre-
a) Incrementa la base monetaria
gada y del producto
b) Incrementa la tasa de descuento
e) Una reducción de la demanda agre-
c) Reduce el nivel del dinero de alto po- gada y un aumento del producto
der
26.En el corto plazo, un aumento per-
d) Todas las anteriores
manente de la oferta monetaria
e) Sucede solo (a) y (b)
a) Tiene mayor impacto en el tipo de
23.Completar el espacio subrayado cambio y el producto que un au-
con una de las siguientes alternati- mento temporal.
vas: “ En el modelo de demanda es- b) Tiene mayor impacto en el tipo de
peculativa de dinero de Tobin la cambio pero menor en el producto
gente mantiene dinero como depósito que un aumento temporal.
de riqueza …………………….”
c) Tiene menor impacto en el tipo de
a) como manera de evitar impuestos cambio y el producto que un au-
b) como una manera de reducir ingre- mento temporal.
sos d) Tiene mayor impacto en el producto
c) como una manera de reducir riesgo pero menor en el tipo de cambio que
d) por todas las razones anteriores un aumento temporal.
e) ninguna de las anteriores e) Ninguna de las anteriores

41 de 84
EXAMEN DE ACTUALIZACION Y SELECCIÓN BCRP
COMPILADOR MARINO ENEQUE

27.¿Cuál de las siguientes afirmacio- e) La suma de las elasticidades de los


nes es verdadera en el corto plazo? volúmenes de exportación e impor-
a) Una expansión monetaria empeora tación debe ser igual a uno para que
la posición de la cuenta corriente de una depreciación ocurra.
la balanza de pagos. 29.Si la tasa de interés en dólares es
b) Una expansión monetaria mejora la 10 por ciento, la tasa de interés en
posición de la cuenta corriente de la euros es 6 por ciento y la deprecia-
balanza de pagos ción esperada del dólar respecto al
c) Una expansión fiscal mejora la posi- euro es 8 por ciento:
ción de la cuenta corriente de la ba- a) Un inversionista debería invertir solo
lanza de pagos. en euros
d) Una expansión fiscal empeora la po- b) Un inversionista debería estar indife-
sición de la cuenta corriente de la ba- rente entre dólares y euros.
lanza de pagos. c) Un inversionista debería invertir solo
e) A y C en dólares.
28.La condición de Marshall – Lerner d) Es imposible responder dada la in-
indica que: formación disponible
a) Una apreciación nominal mejora la e) Todas las anteriores
balanza de cuenta corriente si los 30.En la condición de paridad de tasas
volúmenes de exportación e impor- de interés bajo sustibilidad imperfecta
tación son suficientes elásticos res- de activos
pecto al tipo de cambio real. a) Un aumento de la cantidad de deuda
b) Una depreciación real mejora la ba- pública doméstica aumenta el dife-
lanza de cuenta corriente si los volú- rencial entre el retorno esperado de
menes de exportación e importación los bonos en moneda nacional y ex-
son suficientemente inelásticos res- tranjera.
pecto al tipo de cambio real. b) Una reducción de la cantidad de
c) Una apreciación real mejora la ba- deuda pública doméstica aumenta el
lanza de cuenta corriente si los volú- diferencial entre el retorno espe-
menes de exportación e importación rando de los bonos en moneda na-
son suficientemente inelásticos res- cional y extranjera.
pecto al tipo de cambio real. c) Un aumento de la cantidad de deuda
d) Una depreciación real mejora la ba- pública doméstica reduce el diferen-
lanza de cuenta corriente si los volú- cial entre el retorno esperado de los
menes de exportación e importación bonos en moneda nacional y extran-
son suficientemente elásticos res- jera.
pecto al tipo de cambio real.

42 de 84
EXAMEN DE ACTUALIZACION Y SELECCIÓN BCRP
COMPILADOR MARINO ENEQUE

d) Un aumento de la cantidad de deuda los bonos en moneda nacional y ex-


pública doméstica no afecta el dife- tranjera.
rencial entre el retorno esperado de e) Ninguna de las anteriores

MICROECONOMÍA 2011
31.Dos empresas, 1 y 2, compiten en distancia caminada. Desde el punto
precios. Sus costos marginales son de vista de los vendedores, ¿Cuál es
c1 y c2, respectivamente. Los precios la ubicación optima?
de venta son p1 y p2. Se sabe que el a) Uno a cada extremo de la playa
c1 < c2. b) Los dos al centro de la playa (kilome-
En equilibrio: tro 0.5)
a) p1 = p2 c) Uno en el kilómetro 0.25 y otro en el
b) P1 = c1 0.75, respectivamente
c) P1 = c2 d) Uno al extremo y el otro al centro
d) P1=c2 –e, donde e es un número e) Ningún vendedor debe vender aquí
muy pequeño 34.Una persona tiene la siguiente fun-
32.José y Carla deben escoger entre ción de utilidad: 𝑈 = 𝑑 + 2𝑐 0.5 , donde
micro y macro el próximo ciclo. Si am- d y c se consumen a precios 1 y p,
bos llevan macro, cada uno obtiene 2 respectivamente. El ingreso del indi-
de utilidad. Si ambos llevan micro, viduo es m, donde m<0. Si el indivi-
José obtiene 1 útiles y Carla 2. Si lle- duo es racional, ¿cuál de las siguien-
van los dos cursos distintos, cada tes funciones es correcta?
uno obtiene utilidad 0. ¿cual es el a) Las funciones de demanda son
equilibrio Nash de este juego? c*= 𝑝−2 y d*= m - 𝑝 −1.
a) Los dos llevan macro b) Las funciones son c*= m/p y d*=0.
b) Los dos llevan micro c) Las funciones de demanda son c*=0
c) Llevan cursos distintos y d*=m/p.
d) El juego tiene más de un equilibrio d) a y b son posibles, dependiendo de
Nash la magnitud de m
33.Dos vendedores compiten ven- e) a y c son posibles, dependiendo de
diendo el mismo producto al mismo la magnitud de m
precio en una playa. La playa es una 35.Una firma tiene dos plantas con fun-
línea recta de 1 kilómetro de largo.
ciones de costo c(𝑦1 ) =𝑦12 y c(𝑦2 )
Los bañistas están ubicados de ma-
= 𝑦22 , respectivamente. El producto
nera homogénea y continua a lo largo
total de la firma es y= 𝑦1+ 𝑦2. ¿Cuál
de la playa y caminar les genera una
perdida linealmente proporcional a la

43 de 84
EXAMEN DE ACTUALIZACION Y SELECCIÓN BCRP
COMPILADOR MARINO ENEQUE

es la función de costos de la firma en 38.Considere que X es una variable


términos de y? aleatoria que describe una lotería.
a) c(y) = 𝑦 2 E[x] es el valor esperado y CE [X] es
b) c(y) = 𝑦 /2
2 el equivalente cierto que el individuo
asigna a la lotería. El individuo es ad-
c) c(y) = 2 𝑦 2
verso al riesgo si:
d) c(y) =2𝑦 3
a) CE[X] es siempre mayor que E[x]
e) c(y) =y
b) CE [X] es usualmente mayor, pero a
36.Señale cuál de los siguientes no es veces menor que E[x].
supuesto de un mercado competitivo: c) CE [X] es menor que E[x], siempre
a) muchos consumidores y producto- que X no sea constante.
res, y por tanto agentes tomadores d) CE [X] es usualmente menor que
de precios. E[x], pero a veces mayor que E[x].
b) Movilidad ilimitada de recursos; es Para las dos siguientes preguntas,
decir inexistencia de barreras a la considere la función de utilidad
entrada y salida del mercado.
𝑈(𝑥) = 𝑎𝑥 − (𝑏/2)𝑥 2 , donde a>0
c) Producto heterogéneo: es decir to-
das las firmas producen un producto 39.¿Cuál es la condición que debe sa-
diferenciado. tisfacer el parámetro b para que un
individuo con esta función de utilidad
d) Información completa; es decir fir-
sea adverso al riesgo.
mas y consumidores tienen toda la
información necesaria para tomar a) b<0
decisiones. b) b = 0
e) Ninguna de las anteriores. c) b>0
37.Señale cual afirmación es falsa con d) a y c son correctos.
respecto a un mercado en el cual hay 40.Derive los coeficientes de aversión
una empresa con poder monopólico: al riesgo absoluto (ARA) y el relativo
a) Otras empresas encuentran no ren- (ARR) de esta función de utilidad.
table entra al mercado a) ARA=(b/a-x), ARR=bx/(a-x)
b) Otras empresas encuentran barre- b) ARA=b/(2a-x), ARR=bx/(a-bx)
ras legales para entrar al mercado c) ARA=b/(a-bx), ARR=bx/(a-bx)
c) La empresa que si opera, presenta d) ARA=b/(a-x), ARR=bx/(ab-x)
un costo medio decreciente en un
41.considere un individuo que valora el
amplio rango de niveles de produc-
ocio (1 - L) y el consumo (c) en su
ción
función de utilidad. Considere que el
d) Otras empresas encuentran barre- individuo vive un solo periodo y des-
ras naturales para entrar al mercado
e) Ninguna de las anteriores

44 de 84
EXAMEN DE ACTUALIZACION Y SELECCIÓN BCRP
COMPILADOR MARINO ENEQUE

tina su ingreso laboral (wL) al con- b) La elasticidad de la demanda del


sumo solamente. Dada la función de ofertante X con respecto al precio
preferencias: del ofertante Y es cero.
𝐿1+1/𝜑 c) Una reducción en el precio de uno de
𝑈(𝐶, 𝐿) = log(𝑐) − 𝐵 los ofertantes genera una reacción
1 + 1/𝜑
Donde L denota las horas de trabajo y en el precio/cantidad de todos sus
B el parámetro de preferencias por el competidores.
ocio. Asuma que el individuo decide su d) Las empresas ofertantes enfrentan
nivel de consumo y horas trabajadas demandas perfectamente elásticas.
mediante la maximización de su fun- e) Ninguna de las anteriores.
ción de utilidad sujeto a su restricción 44.considere una industria en la cual
presupuestaria. hay costos marginales decrecientes:
𝜕log(𝐿)
La elasticidad de oferta laboral , i) En equilibrio, el precio debe ser igual
𝜕log(𝑊)
es: al costo marginal
1 ii) En equilibrio, el ingreso marginal
a)
1+1/𝜑 debe ser igual al costo marginal
b) 1+1/𝜑 iii) En equilibrio, habrá muchas empre-
c) 𝜑 sas operando en esta industria
d) 0 iv) En equilibrio, solo habrá una em-
e) Ninguna de las anteriores presa operando en esta industria
42.en el problema anterior, la elastici- a) i) y iii) son correctas
dad de oferta laboral manteniendo b) i) y iv) son correctas
constante la utilidad marginal del con- c) ii) y iii) son correctas
sumo (comúnmente denominado d) ii) y iv) son correctas
elasticidad de oferta laboral “frish”)
e) ninguna de las anteriores.
será:
45.señale cuál de los siguientes en un
a) 1+1/𝜑
bien no rival pero excluyente:
b) 𝜑
a) una manzana
c) Ninguna de las anteriores
b) un parque publico
d) Falta información
c) el aire que respiramos
43.En un mercado oligopólico: d) un programa de computación (soft-
a) El precio es igual al costo marginal. ware)
e) ninguna de las anteriores

45 de 84
EXAMEN DE ACTUALIZACION Y SELECCIÓN BCRP
COMPILADOR MARINO ENEQUE

INGLÈS 2011:

46.Stagflaction occurs when the econ- d) All the above.


omy experiences
a) rising prices and rising output. 48.why do profesional sports player
b) rising prices and falling output. generally earn more than farmers
c) falling prices and falling output. and steelworkers?
d) falling prices and rising output. a) Team owners are monopolists.
b) Sports players are really entertainers
rather than producers .
47.devaluation
c) There are fewer professional sports
a) Refers to an oficial act to alter an ex-
players than farmers or steelwork-
change rate.
ers.
b) Is not required under a system of
completely flexible exchange rates. d) Good sports players are scarcer,
given the demand for their services.
c) Is an alternative to internal adjusta-
ment of an economy under fixed ex-
change rates.

46 de 84
EXAMEN DE ACTUALIZACION Y SELECCIÓN BCRP
COMPILADOR MARINO ENEQUE

EXAMEN DE MACROECONOMÍA 2010


1. Una economía pequeña y abierta a) El Banco central debe igualar la tasa
puede mantener un nivel de deuda de interés a la tasa natural de inte-
positivo en el largo plazo si: rés.
a) La cuenta corriente esta siempre en b) El Banco central debe subir su tasa
equilibrio. de interés en mayor proporción que
b) Existe un flujo de capitales positivo el aumento de las expectativas de
constante. inflación.
c) La cuenta corriente es superavitaria c) EI Banco Central debe usar una re-
en el largo plazo. gla de política monetaria.
d) El producto crece a su nivel poten- d) El instrumento de política monetaria
cial. del Banco Central debe ser la tasa
de interés.
e) Ninguna de las anteriores.
e) Ninguna de las anteriores.
2. En el modelo de crecimiento Robert
Solow: 5. La teoría del consumo de ingreso
permanente implica que:
a) Un incremento en la tasa de ahorro,
eleva la tasa de crecimiento del pro- a) Incrementos transitorios en el nivel
ducto por habitante. de ingreso generan mayor con-
sumo en todos los periodos.
b) En el largo plazo, la tasa de creci-
miento del producto por habitante es b) El nivel de riqueza no afecta el nivel
menor cuando menor es la inver- de consumo.
sión. c) Un aumento del gasto público y una
c) Los ahorros son endógenos, existen reducción de los impuestos tienen
retornos constantes en los factores el mismo efecto sobre la economía.
de producción reproducibles. d) Sólo incrementos transitorios en el
d) El producto por habitante es mayor, ingreso afectan el ahorro.
cuando mayor sea la tasa de aho- e) Ninguna de las anteriores
rro. 6. De los siguientes postulados:
e) Ninguna de las anteriores. I. El producto eficiente es aquél nivel
3. En el mismo modelo de Solow: de actividad económica consistente
a) El salario real crece a la misma tasa con un equilibrio de precios flexi-
que el producto por habitante. bles.
b) La tasa de interés real crece a la II. La tasa natural de desempleo de-
misma tasa que el producto por ha- pende del nivel de inflación de largo
bitante. plazo de la economía.
c) La participación del trabajo en el III. La tasa de referencia de la política
producto es decreciente. monetaria sirve de señal para la de-
terminación de la tasa de interés del
d) La participación del capital en el pro-
mercado interbancario.
ducto es creciente.
Son verdaderos:
e) Ninguna de las anteriores.
a) I y II.
4. El principio de Taylor implica que:
b) II y III.

47 de 84
EXAMEN DE ACTUALIZACION Y SELECCIÓN BCRP
COMPILADOR MARINO ENEQUE

c) SoloI. e) ninguna de las anteriores


d) Solo III. 10.De acuerdo con la Q de Tobin la in-
e) Ninguno es verdadero. versión será mayor cuando:
7. En el modelo Mundell-Fleming, la a) El costo de reposición de la inversión
correlación entre la tasa de interés es mayor.
doméstica y externa será más alta b) Mayor sea la tasa de depreciación
cuando: del capital.
a) Tipo de cambio flexible y no-movili- c) Se espera una menor rentabilidad de
dad de capitales internacionales. la bolsa de valores.
b) Tipo de cambio flexible y perfecta d) La productividad del capital sea me-
movilidad de capitales internaciona- nor.
les. e) Ninguna de las anteriores
c) Tipo de cambio fijo y no-movilidad de 11.De acuerdo con el modelo de Bau-
capitales internacionales. mol. La demanda por dinero será ma-
d) Tipo de cambio fijo y perfecta movili- yor como proporción de los ingresos
dad de capitales internacionales. familiares si:
e) Ninguna de las anteriores a) La volatilidad de la tasa de interés
8. En una economía en donde se cum- sea mayor.
ple la equivalencia Ricardiana. b) Las familias incrementen sus ingre-
Una reducción de impuestos ge- sos.
nera: c) Las familias tengan ingresos bajos.
a) Mayor consumo y producto. d) A y B.
b) Solo mayor producto. e) Todas las anteriores.
c) Menor inversión y consumo. 12.La teoría de la paridad no cubierta
d) Tasas de ahorro privada más bajas. de tasas de interés implica que:
e) Ninguna de las anteriores a) El tipo de cambio sube cuando las
9. La inconsistencia dinámica de la po- tasas de interés domésticas suben.
lítica monetaria implica que: b) El diferencial de tasas de interés do-
a) El Banco Central anuncia como méstica y externan determinan el
meta una inflación más baja, y siem- tipo de cambio forward.
pre la cumple. c) El tipo de cambio es un paseo alea-
b) El Banco central genera mayor infla- torio.
ción y también mayor actividad eco- d) El tipo de cambio se aprecia cuando
nómica. bajan las tasas de interés externas.
c) El Banco Central anuncia una meta e) La clasificación T/NT se modifica
de inflación baja y solo la cumple si drásticamente y el modelo pierde ca-
la actividad económica no es alta. pacidad explicativa.
d) El Banco Central anuncia una meta 13.En un modelo Mundell-Fleming con
de inflación baja, pero tiene el incen- tipo de cambio fijo y perfecta movili-
tivo al no cumplir su promesa, si los dad de capitales un aumento de la
agentes privados creen en el anun- oferta de dinero:
cio.

48 de 84
EXAMEN DE ACTUALIZACION Y SELECCIÓN BCRP
COMPILADOR MARINO ENEQUE

a) Aumenta el producto y baja la tasa e) Ninguna de las anteriores.


de interés 17.Sobre el IPC y el deflactor del PBI
b) Aumenta el producto y aumenta la a) El IPC incluye a todos los bienes y
tasa de interés servicios producidos en el país.
c) Aumenta el producto b) El deflactor del PBI sobreestima el
d) Se reduce la tasa de interés efecto en los consumidores frente al
e) Ninguna de las anteriores incremento en el precio de cierto
14.En un modelo de consumo y ahorro bien.
convencional de dos periodos donde c) El IPC no tiene en cuenta la capaci-
se cumple que para el primer periodo dad de los consumidores de sustituir
Y1= C1+S, siendo y el nivel de ingreso los bienes cuyo precio ha subido.
del individuo, C el consumo y S el d) El deflactor incluye tanto a bienes
ahorro. Además sabemos que en el exportados como importados.
segundo periodo Y2 = C2 + (1+r) S, e) Ninguna de las anteriores
en donde r es la tasa de interés.
18.Sobre los modelos clásico y keyne-
¿Qué efectos tendría en la tasa de in-
siano:
terés un aumento del ingreso en el
periodo 2 únicamente? I. La demanda agregada del modelo
clásico está determinada en forma
a) Aumenta el primer periodo
exclusiva por la cantidad de dinero.
b) Aumenta en ambos periodos
II. En el sistema keynesiano, la de-
c) Se reduce en el periodo 1 manda agregada es una causa im-
d) Se mantiene constante portante de la producción y el em-
e) Ninguna de las anteriores pleo.
15.La curva de Phillips: III. En el sistema keynesiano el dinero
a) Relaciona positivamente la brecha es solo uno de los diversos factores
producto y la inflación. que determinan la demanda agre-
gada.
b) Relaciona negativamente la inflación
y la tasa de desempleo. Son correctas:
c) Tiene pendiente negativa. a. I
d) A y B son verdaderas. b. l y II
e) N.A. c. I y III
d. II y III
16.Los encajes bancarios son:
e. I, II y III
I. Una operación de mercado abierto
del Banco central. 19.Sobre el modelo keynesiano:
II. Se aplican al crédito bancario. I. El efecto multiplicador del gasto de
III. Es una ventanilla de última instan- gobierno es mayor en la medida que
cia. la propensión marginal a ahorrarse
incrementa.
a) I
II. Una acumulación no planeada de
b) I y lI inventarios hará que la producción
c) I y lll tienda a incrementarse.
d) II y III

49 de 84
EXAMEN DE ACTUALIZACION Y SELECCIÓN BCRP
COMPILADOR MARINO ENEQUE

III. La inversión depende de la Q de To- a) Tasas mayores de crecimiento de-


bin. mográfico se traducen en mayores
Son correctas: niveles de producto per cápita.
a. I b) Si una economía tiene más capital
b. I y II que en el estado estacionario de la
regla de oro, una reducción del aho-
c. I y III rro elevará el consumo.
d. II y III c) La tasa de ahorro determina las di-
e. I, Il y III mensiones del stock de capital, y por
20.La movilidad imperfecta de capita- tanto, la tasa de crecimiento de largo
les, entendida bajo el contexto del plazo.
modelo Mundell-Fleming, se origina- d) Una depreciación alta garantiza un
ría debido a que: crecimiento acelerado.
a) La tasa de interés de los activos na- e) Ninguna de las anteriores
cionales no es igual a la tasa de in- 23.La equivalencia Ricardiana implica
terés de los activos extranjeros que:
b) La tasa de interés afecta el nivel de a) El consumo se incrementa si el Go-
consumo bierno baja los impuestos hoy por-
c) Los activos nacionales y extranjeros que planea reducir las compras es-
son sustitutos perfectos tatales futuras.
d) Existen riesgos diferenciales en los b) Financiar el Gobierno mediante
activos de los distintos países y ries- deuda equivale a financiarlo por me-
gos de tipo de cambio dio de impuestos.
e) Ninguna de las anteriores c) El tamaño del Gobierno se incre-
21.En el modelo Mundell-Fleming, con menta con la recaudación.
movilidad imperfecta de capitales d) Más de una es correcta.
frente a una caída exógena de la de- e) Ninguna de las anteriores.
manda de exportaciones y tipo de
cambio flexible: 24.Respecto del consumo la ecuación
de Euler indica:
I. La depreciación del tipo de cambio
compensa al shock negativo a) El nivel de consumo en cada pe-
ríodo.
II. Se incrementa el riesgo político
b) La respuesta del consumo ante cam-
III. Se produce una fuga de capitales bios en el ingreso.
Son correctas: c) La restricción presupuestaria.
a. I d) La senda óptima de consumo.
b . I y lI e) N. A.
c. I y III
25.La Q de Tobin es:
d. II y III
a) El valor actual esperado del costo
e. I, II y III marginal de crear nuevo capital.
22.En un modelo de crecimiento de b) Valor marginal del invertir.
Solow sin cambio tecnológico:

50 de 84
EXAMEN DE ACTUALIZACION Y SELECCIÓN BCRP
COMPILADOR MARINO ENEQUE

c) El valor actual esperado del incre- a) Los agentes nunca se equivocan


mento en el valor de la empresa por por que utilizan de manera eficiente
crear nuevo capital. toda la información relevante.
d) El valor actual esperado el costo me- b) Los agentes nunca se equivocan
dio de crear nuevo capital. porque utilizan de manera eficiente
e) N. A. toda Ia información disponible.
26.¿Para qué valores de la Q de Tobin, c) Aún utilizando toda la información
los inversionistas tendrían incentivos relevante se pueden equivocar.
a incrementar su inversión? d) Aún utilizando eficientemente toda
a) Cuando la Q de Tobin es mayor a la información disponible se pueden
uno. equivocar mas sus errores no son
sistemáticos.
b) Cuando la Q de Tobin es menor a
uno. e) N. A.
c) Cuando la Q de Tobin es mayor al 29.La curva de Laffer relaciona:
costo de reemplazo del capital. a) El nivel de recaudación tributaria y la
d) Cuando la Q de Tobin es mayor al tasa impositiva.
costo de reemplazo al costo margi- b) Cambios en la tasa de desempleo y
nal de ajuste del capital. producto real.
e) N. A. c) Variaciones del tipo de cambio y la
27.¿Cuál de estos eventos no es ca- Balanza comercial.
paz de generar fluctuaciones econó- d) Cambios en el nivel de precios y bre-
micas en un modelo de ciclos reales cha de producto.
de negocios? e) Emisión primaria e inflación.
a) Un incremento inesperado en la pro- 30.Respecto del producto potencial:
ductividad de las empresas. a) Es el nivel de producción que preva-
b) Una reducción inesperada de la tasa lece en una economía si todos los
de interés nominal por parte del precios fuesen flexibles.
banco central. b) Es alterado por fluctuaciones en la
c) Una reducción inesperada del gasto demanda agregada.
real de gobierno. c) Se incrementa cuando el banco cen-
d) Un cambio inesperado de las prefe- tral reduce transitoriamente la tasa
rencias de los agentes. de interés.
e) Un incremento inesperado en la pro- d) No se altera ante cambios en la pro-
ductividad de la inversión. ductividad de las empresas.
28.Sobre el supuesto de expectativas e) Es el nivel de producto que preva-
racionales: lece en una economía si todos los
mercados fuesen de competencia
perfecta.

51 de 84
EXAMEN DE ACTUALIZACION Y SELECCIÓN BCRP
COMPILADOR MARINO ENEQUE

EXAMEN DE MICROECONOMÍA 2010


31.Señale cuál de los siguientes es su- I. La demanda de un bien normal au-
puesto de un mercado competitivo: menta conforme el ingreso del con-
a) Muchos consumidores y producto- sumidor aumenta y por tanto su
res y; por tanto agentes tomadores curva de Engel tendría pendiente po-
de precios sitiva
b) Movilidad ilimitada de recursos es II. La curva de Engel de un bien de lujo
decir inexistencia de barreras a la tiene pendiente positiva pero decre-
entrada y salida del mercado ciente
c) Producto homogéneo es decir todas III. Cuando las preferencias son del
las firmas producen un producto tipo cuasi-lineal, la curva de Engel es
idéntico una recta vertical
d) Información completa; es decir fir- a) VVV
mas y consumidores tienen toda la b) VVF
información necesaria para tomar c) FFV
decisiones d) VFV
e) Todas las anteriores e) FFF
32.Considere una función de produc- 34.Cuando el monopolista no puede
ción Cobb Douglas con 2 factores de distinguir entre las demandas de los
producción – capital (K) y trabajo (L) distintos consumidores tendrá que
- de la siguiente forma: ofrecer paquetes tarifarios; y los con-
F ( K , L)  K  L , α+β=1 sumidores voluntariamente Y en fun-
ción de sus propias curvas de de-
Esta función presenta: manda elegirán el que más le con-
a) Retornos a escala constante y elas- venga A. esta práctica se le deno-
ticidad de sustitución entre factores mina:
menor a 1 a) Discriminación de primer grado
b) Retornos a escala creciente y elasti- b) Discriminación de segundo grado
cidad de sustitución entre factores
menor a 1 c) Discriminación de tercer grado
c) Retornos a escala constante y elas- d) Discriminación de cuarto grado
ticidad de sustitución entre factores e) Ninguna de las anteriores
igual a 1 35.En el modelo oligopólico de Ber-
d) Retornos a escala creciente y elasti- trand:
cidad de sustitución entre factores a) Las empresas compiten por cantidad
igual a 1 dentro de un esquema de juego si-
e) Retornos a escala decreciente elas- multáneo
ticidad de sustitución entre factores b) Las empresas compiten por cantidad
menor a 1 de en forma de juego secuencial,
33.Determine si los siguientes enun- donde el líder mueve primero
ciados son verdaderos o falsos. c) Las empresas compiten por precio
en forma simultánea

52 de 84
EXAMEN DE ACTUALIZACION Y SELECCIÓN BCRP
COMPILADOR MARINO ENEQUE

d) Las empresas compiten por precio 39.El Lema de Shephar indica que:
en forma secuencial donde el líder
a) La derivada parcial de la función de
mueve primero gasto con respecto al precio del bien
e) Las empresas se ponen de acuerdo es igual a la demanda Hicksiana de
en eI precio que van a cobrar por el dicho bien.
producto
b) La derivada parcial de la función de
36.Se define como bienes públicos a utilidad máxima con respecto al pre-
aquellos que son: cio del bien es igual a la demanda
a) No rivales y exclusivos Marshalliana de dicho bien.
b) Rivales y exclusivos c) Las funciones de demanda obteni-
c) Rivales y no exclusivos das de una maximización de la utili-
d) No rivales y no exclusivos dad sujeta a una restricción presu-
puestaria son iguales a las funciones
e) Ninguna de las anteriores de demanda obtenidas de una mini-
37.Respecto a las demanda mars- mización de costo sujeto a un nivel
halliana y hicksiana es cierto que: predeterminado de utilidad.
a) Ante un cambio en los precios, la va- d) Las funciones de demanda obteni-
riación equivalente muestra cuánto das de una maximización de la utili-
hay que otorgar o quitar al consumi- dad sujeta a una restricción presu-
dor para que a los precios finales dis- puestaria son iguales a las funciones
frute del bienestar que tenía con los de demanda obtenidas de una mini-
precios iníciales. mización de costo sujeto a un nivel
b) Ante un cambio en los precios, la va- predeterminado de utilidad.
riación compensada muestra cuanto e) Ninguna de las anteriores.
ingreso habría que otorgar o quitara 40.¿Cuál de las siguientes afirmacio-
un consumidor para quea los precios nes es cierta acerca de los bienes Gi-
iniciales disfrute del bienestar que ffen?
tiene con los precios finales
a) Solamente si el ingreso aumenta y
c) Si el bien es inferior, la demanda los precios aumentan, su consumo
marshalliana es menos elástica que aumenta.
la hicksiana
b) Solamente si el ingreso disminuye y
d) En algunos casos la demanda hi- los precios aumentan, su consumo
cksiana puede ser creciente en su disminuye.
precio
c) Si los precios disminuyen su con-
e) Ninguna de las anteriores sumo disminuye.
38.¿Cuál de las siguientes funciones d) Solamente si su elasticidad cruzada
de utilidad esperada indican un com- es negativa y su elasticidad ingreso
portamiento amante al riesgo? es cero, su consumo cae si los pre-
a) U(x) = x cios aumentan.
b) U(x) = 1/x e) Ninguna de las anteriores
c) U(x) = 22x 41.En una economía de dos bienes si
d) U(x) = log(x) un bien es inferior entonces:
e) U(x) = (x+5)2 a) El otro debe ser de lujo.

53 de 84
EXAMEN DE ACTUALIZACION Y SELECCIÓN BCRP
COMPILADOR MARINO ENEQUE

b) El otro puede ser inferior. b) Información perfecta.


c) Ambos bienes deben ser comple- c) Carencia de barreras a la entrada o
mentarios. salida.
d) El otro debe ser normal. d) No hay costos de transacción.
e) Ambos bienes son sustitutos. e) Ninguna de las anteriores.
42.Por favor indique el equilibrio de 44.En el Dilema del Prisionero, el equi-
Nash del siguiente juego: librio de Nash (delatar, delatar) no
será alcanzado si:
a) Si es un juego de una vez.
Seguir No seguir
b) si es un juego repetido un número fi-
Seguir (200, 200) (-100,100) nito de veces.
No seguir (100,-100) (-500,-500) c) Si es un juego repetido infinitas ve-
a) (Seguir, seguir) ces.
b) (Seguir, No Seguir) d) Todas las anteriores.
c) (No Seguir, Seguir) e) Ninguna de las anteriores.
d) (No Seguir, No Seguir) 45. Una función de utilidad es homoté-
tica si:
e) No hay equilibrio de Nash
a) La curva de Engel es lineal
43.Por favor indique cuál de las si- b) La curva de Engel es cuadrática
guientes afirmaciones No es un su-
c) La curva de Engel es decreciente
puesto del modelo de competencia
d) El gasto es constante.
perfecta.
e) Ninguna de las anteriores
a) Bienes homogéneos.

EXAMEN DE INGLES 2010

Lea, por favor, el siguiente extracto del Capítulo I del libro de Frederic S. Mishkin
(2009), "Economics of Money, Banking, and Financia Mlarkets" y responda las
preguntas que luego se le plantean al respecto.
The topic of money, banking, and financial markets is an exciting field that directly
affects your life-interest rates influence earnings on your savings and the pay-
ments on loans you may seek on a car or a house, and monetary policy may
affect your job prospects and the prices of goods in the future. Your study of
money, banking, and financial markets will introduce you to many of the, contro-
versies about the conduct of economic policy that are currently the subject of hot
debate in the political arena and will help you gain a clearer understanding of
economic phenomena you frequently hear about in the news media.The
knowledge you gain will stay with you and benefit you long after the course is
done.

54 de 84
EXAMEN DE ACTUALIZACION Y SELECCIÓN BCRP
COMPILADOR MARINO ENEQUE

46.A fall in interest rates might af- c) Providing you with savings and
fect your economic behavior by: loans.
d) Providing you with knowledge about
a) Increasing your earnings on savings.
economic phenomena.
b) Reducing your earnings on savings.
e) Protecting you against the news me-
c) Increasing your earnings on cars. dia.
d) Reducing your earnings on cars.
e) Neither reducing nor increasing ex- 48.What does the term hot refers to?
pending. a) The political arena is warm.
47.What is basic activity of banks? b) Economic phenomena is wild.
c) Money and banking are important.
a) Introducing you to many controver-
d) Intense discussion
sies.
b) Introducing you to the political arena. e) Banking is an exciting field

55 de 84
EXAMEN DE ACTUALIZACION Y SELECCIÓN BCRP
COMPILADOR MARINO ENEQUE

BCRP CURSO DE ACTUALIZACION Y SELECCION 2009


EXAMEN DE CONOCIMIENTOS GENERALES
1. ¿Cuál de los siguientes países no ha c. 2,5 por ciento, con un rango de +/-1
adoptado el euro como moneda? punto porcentual
a. Eslovaquia d. 2 por ciento
b. Dinamarca e. Ninguna de las anteriores
c. Alemania 6. Señala cuál de las siguientes siglas
d. Francia no hace referencia a un acuerdo de
e. Portugal integración económica entre países
de este continente:
2. Las siglas del APEC corresponden
a. NAFTA
a:
b. CAN
a. Asian-Pacific Economic Camara
c. MERCOSUR
b. Asian-Pacific Economic Countries
d. SACU
c. Asian-Pacific Economic Cooperation
e. Todas hacen referencia a acuerdos
d. Asian-Pacific Economic Center
regionales en América.
e. Asian-Pacific Economic Community
7. Señale cuál de las siguientes perso-
3. Señale en qué rango se ubicó el cre- nas no ha recibido el Premio Nobel de
cimiento económico del Perú en el Economía.
año 2008:
a. Paul Krugman
a. 2-4 por ciento
b. Joseph Stiglitz
b. 4-6 por ciento
c. Kenneth Rogoff
c. 6-8 por ciento
d. Kenneth Arrow
d. 8-10 por ciento
e. Todos los anteriores han recibido el
e. Ninguna de las anteriores Premio Nobel.
4. Señale cuál de los siguientes even- 8. Diga entre qué rango se ha venido
tos NO está relacionado con la actual cotizando en las últimas semanas el
crisis financiera internacional. euro respecto al dólar.
a. Quiebra de Lehman Brothers a. Entre 1,3 y 1,5 dólares por euro.
b. Rescate de AIG b. Entre 0,8 y 1,0 dólares por euro.
c. Quiebra del Long-Term Capital Man- c. Por debajo de un dólar por euro.
agement (LTCM)
d. Por encima de 2 dólares por euro.
d. El lanzamiento del programa TARP
e. Se ha mantenido fijo en 1,11 dólares
e. Todas las anteriores. por euro desde junio de 2008.
5. La meta de inflación del Banco Cen- 9. Señale cuál de las afirmaciones no
tral de Reserva del Perú es: es correcta:
a. 2 por ciento, con un rango de +/-1 a. México es un país miembro de la
punto porcentual OECD.
b. 3 por ciento b. Noruega no es miembro de la OPEP.

56 de 84
EXAMEN DE ACTUALIZACION Y SELECCIÓN BCRP
COMPILADOR MARINO ENEQUE

c. Perú es uno de los países miembros e. Todas han sido optadas.


de la OMC. 13.El país que tiene el mayor monto de
d. Dominique Strauss-Kahn es actual- bonos del tesoro de Estados Unidos
mente el director ejecutivo del FMI. es:
e. Todas las anteriores son correctas. a. China
10.Ordene, de mayor a menor, las ta- b. Japón
sas de interés de los siguientes ban- c. Reino Unido
cos centrales: d. Rusia
a. Perú, Brasil, Estados Unidos. e. No se sabe.
b. Brasil, Estados Unidos, Perú
14.Señale cuáles son los dos produc-
c. Perú, Estados Unidos, Brasil tos de exportación más importantes
d. Brasil, Perú, Estados Unidos (en términos de valor) de Perú:
e. Estados Unidos, Perú, Brasil a. Cobre y plata.
11.La finalidad del Banco Central de b. Oro y cobre.
Reserva del Perú es: c. Cobre y harina de pescado.
a. Preservar la estabilidad monetaria. d. Páprika y cobre.
b. Promover el crecimiento. e. Cobre y zinc.
c. Lograr el pleno empleo. 15.Hasta diciembre de 2001, previo a
d. a. y b. la crisis argentina, el dólar se cam-
e. Ninguna de las anteriores. biaba por un peso. Pocas semanas
12.Señale cuál de las siguientes medi- después, un dólar se cambiaba por
das NO ha sido adoptada por el cuatro pesos. Ello significa que:
BCRP desde la agudización de la cri- a. El dólar se apreció 300% con res-
sis financiera internacional (setiembre pecto al peso.
de 2008). b. El peso de depreció 300% respecto
a. Reducción de la tasa de Interés. al dólar.
b. Reducción de la tasa de encaje. c. El peso se depreció 75% respecto al
c. Aumento de los plazos de las opera- dólar.
ciones de liquidez del BCRP. d. a y c.
d. Compra de bonos del gobierno y del e. a y b.
sector privado.

EXAMEN DE APTITUD MATEMÁTICA 2009


16.En un grupo de 15 estudiantes, 7 c. 4
han estudiado inglés, 8 francés y 3 no d. 5
han estudiado idiomas. ¿Cuántos e. 7
alumnos han estudiado ambos idio-
mas, inglés y francés? 17.Si x 2 + 2xy + y 2 = 25, entonces:
a. 0 a. x 2 + y 2 > (x + y)2
b. 3 b. x 2 + y 2 < (x + y)2

57 de 84
EXAMEN DE ACTUALIZACION Y SELECCIÓN BCRP
COMPILADOR MARINO ENEQUE

c. x 2 + y 2 = (x + y)2 ¿Qué parte del césped quedó sin cor-


d. N.A. tar?
2−𝑋
e. No se puede determinar. a.
𝑋
18.Si la operación 0 es definida por la b.
𝑋
ecuación: x 0 y = 2x + y, ¿cuál es el 2
valor de a en la ecuación 2 0 a = c. 𝑋 − 2
𝑋−2
a 0 3? d.
2
a. 0 𝑋−2
e.
b. -1 𝑋
c. 1 23.Los ingresos de un mesero constan
d. 1,5 de un salario se S/. 50 a la semana y
propinas. Durante una semana, sus
e. 4 propinas fueros 5/4 de su salario.
19.Si n y p son números impares, ¿Qué fracción de sus ingresos a la
¿cuál de los siguientes números será semana provino de propinas?
siempre un número par? 4
a.
a. 𝑛𝑝 9
1
b. 𝑛 + 𝑝 b.
2
c. 𝑛𝑝 + 2 c.
5
9
d. 𝑛 + 𝑝 + 1 5
d.
e. 2𝑛 + 𝑝 8
5
20.Si A > 𝐵 y B > 𝐶, entonces: e.
4
a. 𝐴 > (𝐵 + 𝐶) 24. La llanta de un automóvil tiene 2
b. 2𝐴 < (𝐵 + 𝐶) huecos. El primer hueco puede desin-
c. 2𝐴 > (𝐵 + 𝐶) flar la llanta totalmente en 9 minutos.
d. 2𝐴 = (𝐵 + 𝐶) El segundo hueco puede desinflar la
llanta totalmente en 6 minutos.
e. No se puede determinar. ¿Cuánto tiempo tomará a los 2 hue-
21.Un hombre recorre d kilómetros en cos juntos desinflar la llanta? Asuma
t horas. ¿A qué ratio (en horas) puede que el aire sale de la llanta a una tasa
recorrer m kilómetros? constante.
a. 𝑑𝑚𝑡 3
a. 3 minutos
𝑚𝑑 5
b. b. 4 minutos
𝑡
𝑚𝑡
c. 1
c. 5 minutos
𝑑 4
𝑑𝑡 1
d. d. 7 minutos
𝑚 2
e.
𝑑
e. 15 minutos
𝑡
22.Una mujer puede cortar su césped 25.En un hexágono regular, ¿cuál es
en X horas. Después de 2 horas de la suma de los ángulos interiores?
iniciado el trabajo empieza a llover. a. 900
b. 720

58 de 84
EXAMEN DE ACTUALIZACION Y SELECCIÓN BCRP
COMPILADOR MARINO ENEQUE

c. 540 a. 80
d. 360 b. 160
e. 180 c. 90
26.En un parque, la relación de juga- d. 45
dores de fútbol respecto de jugadores e. 40
de voleibol es de 3 a 4. Si el total de 29.Un asaltante después de robar un
jugadores suma 63, ¿en cuánto ex- banco, huye con el botín en un auto a
cede el número de jugadores de vo- razón de 80 km/h. Un policía empieza
leibol al número de jugadores de fút- a perseguirlo después de 15 minutos.
bol? ¿A qué velocidad viajó el policía, si lo
a. 3 capturó después de 50 minutos de
b. 4 persecución?
c. 6 a. 110 km/h
d. 9 b. 120 km/h
e. 12 c. 100 km/h
27.Si el largo de un rectángulo se in- d. 104 km/h
crementa en 20 por ciento y el ancho e. 160 km/h
del mismo se reduce en 20 por ciento, 30.Cuando se echan 6 galones de ga-
entonces el área del rectángulo: solina a un auto, el indicador de gaso-
a. Se reduce en 20 por ciento. lina sube de ¼ a 5/8. La capacidad to-
b. Se reduce en 4 por ciento tal del tanque de gasolina (en galo-
c. Permanece invariable. nes) es:
d. Aumenta en 20 por ciento. a. 12
e. Aumenta en 40 por ciento. b. 14
28.Dos números enteros son entre sí c. 15
como 10 es a 9. Si la suma de la mitad d. 16
del mayo y la tercera parte del menor e. 30
es 72. Hallar el mayor de los dos nú-
meros.

59 de 84
EXAMEN DE ACTUALIZACION Y SELECCIÓN BCRP
COMPILADOR MARINO ENEQUE

EXAMEN DE MACROECONOMÍA 2009

31.En el modelo de crecimiento de a. La introducción de un sistema de se-


Ramsey: guridad social puede tener un efecto
negativo o positivo sobre el ahorro
a. Los ahorros son exógenos, y existen total de economía, dependiendo de
retornos decrecientes en los factores la forma en que este sistema sea in-
de producción reproducibles troducido
b. Los ahorros son exógenos, existen b. La introducción de un sistema de se-
retornos constantes en los factores guridad social no tiene efecto alguno
de producción reproducibles sobre el ahorro total de la economía
c. Los ahorros son endógenos, existen c. La introducción de un sistema de se-
retornos decrecientes en los factores guridad social tendrá un efecto nega-
de producción reproducibles tivo sobre el ahorro total de la econo-
d. Los ahorros son endógenos, existen mía
retornos constantes en los factores d. La introducción de un sistema de se-
de producción reproducibles guridad social tendrá un efecto posi-
e. Ninguna de las anteriores tivo sobre el ahorro total de la econo-
mía
32.En la política monetaria existen dos
factores básicos que el diseñador de e. Ninguna de las anteriores
política debe considerar: credibilidad 34.La Equivalencia Ricardiana esta-
y habilidad de responder a choques blece que:
de oferta. En este contexto:
a. La política monetaria es neutral, es
a. Una política monetaria discrecional decir que no tiene ningún efecto so-
es creíble, pero no provee flexibilidad bre las variables reales, y por tanto
al diseñador de política para respon- no tiene efecto sobre la economía
der a choques de oferta
b. La política monetaria es superneu-
b. Una política monetaria con una meta tral, es decir que no tiene ningún
fija no es creíble, pero provee flexibi- efecto sobre las variables reales a
lidad al diseñador de política para excepción de los saldos reales de di-
responder a choques de oferta nero
c. Una política monetaria con una meta c. Un aumento del gasto público y una
fija es creíble, pero no provee flexibi- reducción de los impuestos tienen el
lidad al diseñador de política para mismo efecto sobre la economía
responder a choques de oferta
d. Solo la cantidad de las compras del
d. Una política monetaria discrecional gobierno, y no la distinción entre los
no es creíble y no provee flexibilidad tipos de financiamiento para hacer
al diseñador de política para respon- esas compras, afectan la economía.
der a choques de oferta
e. Ninguna de las anteriores
e. Ninguna de las anteriores
35.De los siguientes postulados:
33.En el modelo de generaciones tras-
lapadas:

60 de 84
EXAMEN DE ACTUALIZACION Y SELECCIÓN BCRP
COMPILADOR MARINO ENEQUE

I. La condición Marshall Lerner esta- b. Reducir el gasto público


blece que una devaluación tendrá un c. Depreciar la moneda doméstica
efecto positivo sobre la balanza co- d. Aumentar la tasa de interés
mercial, si la suma de las elasticida-
des de las exportaciones e importa- e. Ninguna de las anteriores
ciones es igual a 1 en valor absoluto 38.Si los coeficientes de passthrough
II. En los modelos de crisis cambiaria de las precios de importaciones son
de segunda generación, los ataques bajos, entonces:
especulativos pueden a aparecer in-
cluso cuando no existan problemas a. La meta inflación es el régimen de
con los “fundamentos económicos” política monetaria más efectivo
III. El modelo de Overshooting de b. La meta inflación es el régimen de
Dornbusch se basa en la rigidez de política monetaria menos efectivo
los precios en el mercado de bienes c. La meta tipo de cambio es el régimen
y el rápido ajuste del mercado finan- de política monetaria más efectivo
ciero. d. La meta tipo de cambio es el régimen
Son verdaderos: de política monetaria menos efectivo
a. I y II e. Ninguna de las anteriores
b. II y III 39.La medición de la inflación usando
c. Solo I la variación en un índice de precios
d. Solo III tipo Laspeyres genera una sobre-es-
e. Ninguno es verdadero timación de la inflación que puede lle-
gar a ser más de 1 por ciento. Esta
36.En el modelo Mundell-Fleming, el sobre-estimación NO se explica por:
banco central no puede controlar la
oferta de dinero bajo las siguientes a. La exclusión del efecto sustitución
circunstancias: entre bienes
b. El uso de ponderados fijos
a. Tipo de cambio flexible y no-movili-
c. La exclusión del efecto de descuen-
dad de capitales internacionales
tos en las grandes tiendas
b. Tipo de cambio flexible y perfecta
d. La exclusión del efecto de mayor
movilidad de capitales internaciona-
precio por mayor calidad
les
e. Ninguna de las anteriores
c. Tipo de cambio fijo y no-movilidad de
capitales internacionales 40.En el modelo de bienes y servicios
d. Tipo de cambio fijo y perfecta movili- transables (T) y no transables (NT),
dad de capitales internacionales los determinantes principales de esta
e. Ninguna de las anteriores clasificación de bienes y servicios
son:
37.Un país importador de combusti-
bles enfrenta un alto déficit en cuenta a. Los costos de transporte desde los
corriente. ¿Cuál de las siguientes al- puertos nacionales hacia el exterior
ternativas NO reducirá este déficit? b. Los costos de descarga y embalaje
en los puertos nacionales
a. Reducir impuestos

61 de 84
EXAMEN DE ACTUALIZACION Y SELECCIÓN BCRP
COMPILADOR MARINO ENEQUE

c. Los impuestos a las importaciones efectos tendría en el consumo del pe-


(aranceles) y los subsidios a las ex- riodo 1 y 2 ante un aumento de la tasa
portaciones de interés?
d. A y C a. Se reducen en ambos periodos
e. Todas las anteriores b. Aumenta en ambos periodos
41.En el reciente proceso de globaliza- c. Se reduce el consumo del periodo 1
ción se ha producido una continua y aumenta en el periodo 2
disminución de los costos de trans- d. Se mantienen constantes
porte así como de los aranceles. e. Ninguna de las anteriores
¿Cuál puede ser el mayor impacto en
el modelo de bienes y servicios tran- 44.La curva de Laffer:
sables (T) y no transables (NT)?
a. Relaciona la tasa impositiva prome-
a. El precio relativo PT/PNT. dio y el nivel de ingresos recauda-
b. La combinación óptima de produc- dos por el gobierno
ción QT /QNT . b. Se reduce a medida que aumenta la
c. El consumo relativo CT /CNT tasa impositiva promedio
d. La balanza comercial c. Tiene pendiente negativa
e. La clasificación T/NT se modifica d. No tiene máximo
drásticamente y el modelo pierde e. N.A.
capacidad explicativa
45.Bajo la equivalencia Ricardiana, se
42.En un modelo Mundell - Fleming cumple que:
con tipo de cambio fijo y perfecta mo-
I. Los agentes son racionales
vilidad de capitales y en el cual se
cumple la condición Marshall-Lerner, II. No hay efectos reales de una reduc-
un aumento de la oferta de dinero ción de impuestos
tiene efectos en: III. Puede no cumplirse si existen res-
tricciones de liquidez.
a. Aumenta el producto y baja la tasa
a. I
de interés
b. I y II
b. Aumenta el producto y aumenta la
tasa de interés c. I y III
c. Aumenta el producto d. II y III
d. Se reduce la tasa de interés e. I, II y III
e. Ninguna de las anteriores 46.¿Por qué no se cumple la Paridad
del Poder de Compra (PPP)?
43.En un modelo de consumo y ahorro
convencional de dos periodos, donde a. Estandarización de productos
se cumple que para el primer periodo b. Impuestos
Y1 = C1 + S, siendo Y el nivel de in- c. Diferencias en costos laborales
greso del individuo, C el consumo y S d. Más de una es correcta
el ahorro. Además sabemos que en el e. Ninguna es correcta
segundo periodo Y2 = C2 + (1 + r)S ,
en donde r es la tasa de interés. ¿Qué

62 de 84
EXAMEN DE ACTUALIZACION Y SELECCIÓN BCRP
COMPILADOR MARINO ENEQUE

47.Según el modelo Mundell-Fleming b. Las políticas que incentivan el aho-


para una economía abierta, la elec- rro, no generan crecimiento de largo
ción del régimen cambiario de un país plazo.
depende: c. La convergencia β señala que paí-
ses con menor ingreso tienden a cre-
a. Del tipo de choque al que la econo-
cer más rápido en términos per-cá-
mía está expuesta
pita.
b. De la independencia del banco cen-
tral d. Bajo la hipótesis de convergencia
c. De la profundidad del mercado de condicional, puede ocurrir que los
capitales países ricos crezcan más rápido que
d. De la brecha del producto y el nivel los países pobres.
de desempleo e. Todas las anteriores.
e. Todas las anteriores
50.Sobre los modelos de crecimiento:
48.Sobre la función de consumo key-
a. Es posible tener crecimiento a largo
nesiana:
plazo, cuando se incluye el concepto
a. La propensión media a consumir es de externalidades en la función de
constante. producción neoclásica.
b. La tasa de interés afecta el nivel de b. En los modelos de crecimiento endó-
consumo. geno, la solución del mercado siem-
c. Se corrobora con la evidencia empí- pre es superior a la de un planifica-
rica para el corto plazo. dor centralizado.
d. El consumo es principalmente afec- c. El concepto de “creación destruc-
tado por el ingreso permanente. tiva” está asociado al modelo neo-
e. Ninguna de las anteriores. clásico de Ramsey.
d. Schumpeter desarrolló el primer mo-
49.En el modelo neoclásico de Solow: delo de crecimiento exógeno.
a. Solo con un choque tecnológico exó- e. Ninguna de las anteriores.
geno se obtiene crecimiento de largo
plazo.

EXAMEN DE MICROECONOMÍA 2009

51.Un consumidor desea desplazarse a. Si los precios de ambos bienes coin-


a su puesto de trabajo y puede elegir ciden, el consumidor podrá elegir
entre viajar en tren o autobús. Si al in- cualquier combinación de ambos
dividuo le resulta indiferente cual- bienes siempre que cumpla su res-
quiera de estas dos opciones y la tricción presupuestaria
elasticidad cruzada de la demanda b. Si el precio del tren es superior al del
entre ambos bienes es positiva, se- autobús, el consumidor estará en un
ñale la respuesta correcta: óptimo gastando toda su renta en el
tren

63 de 84
EXAMEN DE ACTUALIZACION Y SELECCIÓN BCRP
COMPILADOR MARINO ENEQUE

c. Independientemente de cuáles sean b. Disminuirá los beneficios porque el


los precios de los bienes, al ser bie- costo marginal a corto plazo será
nes complementarios optimizará su menor que el precio.
situación consumiendo la misma c. Aumentará los beneficios porque el
cantidad de ambos bienes costo marginal a corto plazo será
d. Independientemente de cuáles sean menor que el precio
los precios de los bienes, al ser sus- d. Disminuirá los beneficios porque el
titutivos siempre estará optimizando costo marginal a corto plazo será
cuando se encuentre en una solu- mayor que el precio
ción esquina e. Ninguna de las anteriores
e. Ninguna de las anteriores
54.Suponga que los consumidores del
52.Partiendo de una situación de equi- tipo A tienen una función de demanda
librio, donde la cantidad y el precio de inversa que viene dada por Pa = 20 −
equilibrio son 20 y 100 respectiva- 4Qa y los del tipo B tienen una función
mente. Al aumentar el precio en 10 de demanda inversa que viene dada
unidades por el efecto sustitución la por Pb = 10 − 4Qb . Si la industria
cantidad disminuirá en 3 unidades. Si competitiva produce un total de 10
la pendiente de la función de de- unidades, ¿cuánto se compraría en
manda indica que el precio aumenta cada mercado?
en 10 unidades al disminuir la canti-
dad en 1 unidad, entonces: a. Qa = 4, Qb = 6
b. Qa = 3, Qb = 7
a. El efecto ingreso será negativo indi-
cando que se trata de un bien inferior c. Qa = 5, Qb = 5
b. El efecto ingreso será positivo indi- d. Qa = 7, Qb = 3
cando que se trata de un bien normal e. Ninguna de las anteriores
c. Para que la pendiente de la función 55.Una empresa de software que ac-
de demanda sea negativa, obligato- túa como monopolista de sus progra-
riamente el efecto ingreso debe ser mas informáticos debe fijar los pre-
positivo cios en dos mercados. El mercado 1
d. El efecto ingreso siempre es nega- está compuesto por las empresas y el
tivo es una función de demanda de- Estado, el mercado 2 está compuesto
creciente por estudiantes. La empresa sabe
e. Ninguna de las anteriores que existe el riesgo de piratería en el
mercado uno es -1 y en el mercado 2
53.Si una empresa competitiva está es -4, entonces:
produciendo en el corto plazo en el
punto donde el costo marginal es a. Aplicará el mismo precio en ambos
igual al precio siendo el costo margi- mercados
nal creciente, un incremento de la b. Aplicará un precio más bajo en el
producción: mercado 1
a. Aumentará los beneficios porque el c. Fijará un precio más bajo en el mer-
costo marginal a corto plazo será cado 2
mayor que el precio

64 de 84
EXAMEN DE ACTUALIZACION Y SELECCIÓN BCRP
COMPILADOR MARINO ENEQUE

d. Con esta información, no podemos b. 25


afirmar nada sobre su política de c. 30
precios óptima d. No es posible calcular el precio sin
e. Ninguna de las anteriores conocer la curva de demanda
56.Suponga que un individuo que con- e. Ninguna de las anteriores
sume C1 en el periodo 1 y C2 en pe- 59.En un mercado en competencia
riodo 2, mientras que su ingreso es perfecta, no es correcto que:
m1 y m2 en cada periodo y la tasa de
interés es i. Entonces si en el periodo a. En equilibrio de largo plazo, las dife-
1 no consume nada la cantidad má- rentes empresas terminan operando
xima que puede consumir en periodo cada una en el punto mínimo de su
2 es: costo medio de largo plazo
𝐶 b. La oferta de cada empresa, en el
a. 1 corto plazo, es su curva de costo
1+𝑖
𝑚
b. 2 marginal, a partir del punto mínimo
1+𝑖
𝑚1 +𝑚2
de su costo medio variable
c. c. El número de empresas operando
1+𝑖
d. (1 + 𝑖)𝑚1 + 𝑚2 en equilibrio de largo plazo es deter-
e. Ninguna de las anteriores minado por la magnitud de la de-
manda y la cantidad ofertada por
57.En el contexto de un modelo inter- cada empresa en el punto mínimo
temporal y partiendo de una situación del costo medio de largo plazo
donde el individuo inicialmente estu- d. Cada empresa elige ofertar en el
viese ahorrando y el gobierno decide punto en el cual el precio de mer-
aplicar un impuesto sobre el ingreso cado prevaleciente es igual a su
futuro. El efecto de esta política será: costo marginal
a. Un desplazamiento paralelo de la e. Ninguna de las anteriores
restricción presupuestaria hacia el
60.En el negocio de los seguros contra
origen accidentes automovilísticos, es inco-
b. Si los bienes de consumo en el pe- rrecto afirmar que:
riodo 1 y 2 fuesen normales, dismi-
nuirían las cantidades consumidas a. Existe un problema de información
c. Aumentaría el ahorro del individuo asimétrica ya que los asegurados
cuentan con mayor información que
d. Todas las respuestas anteriores son los aseguradores con respecto a su
correctas probabilidad de tener un accidente
e. Ninguna de las anteriores b. Las firmas aseguradoras siempre
58.Suponga que un monopolista puro ofertarán cobertura cobrando en fun-
que está produciendo en un punto ción a una probabilidad de choque
donde E = −3, si el costo marginal es por encima del promedio observado
constante e igual a 10, el precio al que en toda la población relevante
vende el producto es igual a: c. El seguro contra accidentes automo-
vilísticos será acogido por solo una
a. 15 fracción de la población relevante,

65 de 84
EXAMEN DE ACTUALIZACION Y SELECCIÓN BCRP
COMPILADOR MARINO ENEQUE

representando los demás aquella los conductores mostrarán una me-


parte de la población que deja de ser nor preocupación al manejar sa-
atendida debido al problema de se- biendo que el seguro los cubre
lección adversa e. Ninguna de las anteriores
d. Hay un problema de riesgo moral
una vez contraído el seguro, ya que

66 de 84
EXAMEN DE ACTUALIZACION Y SELECCIÓN BCRP
COMPILADOR MARINO ENEQUE

BCRP EXAMEN DEL CURSO DE ACTUALIZACIÓN 2008


MACROECONOMÍA III. Una caída en 10 del gasto público
1. Señale la condición verdadera: incrementa el nivel de precios en 5
a. Señale la opción de “arbitraje de ta- IV. Si la oferta agregada cambia a
sas de interés” establece que la tasa 𝑄 𝑆 = 15 + 3𝑃, el nivel de equilibrio
de interés doméstica (en moneda lo- de precios caería de 35 a 15
cal) es igual a la tasa de interés ex-
terna más la tasa de depreciación a. Sólo I y II son verdaderas.
de la moneda b. Sólo II y IV son verdaderas.
b. El término “overshooting” del tipo de c. Sólo II, III y IV son verdaderas.
cambio se refiere a la respuesta más d. Sólo II y III son verdaderas.
que proporcional del tipo de cambio e. Ninguna de las anteriores.
experimenta ante un choque mone-
tario 4. Señale a opción verdadera:
c. La versión relativa de la teoría de la a. La curva IS se desplaza hacia la de-
paridad de poder de compra implica recha ante una disminución de la
que la inflación interna es igual a la tasa de interés.
tasa de depreciación de la moneda b. El efecto desplazamiento o
más la tasa de inflación externa “Crowding out” es nulo si la curva IS
d. Todas las anteriores es horizontal
e. Ninguna de las anteriores c. La curva LM se desplaza hacia la de-
recha cuando el nivel de precios cae
2. La brecha producto se define como: d. Solo b y c
a. El nivel del producto por encima del e. Ninguna de las anteriores
producto potencial
b. El nivel del producto que equilibra el 5. La curva LM define combinaciones
mercado de bienes entre una tasa de interés y producto
c. El nivel del producto que genera pre- consistente con equilibrio en
siones de demanda a. El mercado de bienes
d. El nivel del producto consistente b. El mercado de divisas
como inflación cero c. El mercado de dinero
e. Ninguna de las anteriores d. El mercado laboral
e. Ninguna de las anteriores
3. Considere el siguiente modelo:
𝐶 = 40 − 𝑃 6. Una expansión de la curva IS esta-
𝐼 = 20 ría más probablemente asociada con
𝐺 = 15 a. Una reducción de impuestos
𝑄𝑆 = 5 + 𝑃 b. Una reducción del gasto guberna-
Donde C es el Consumo Privado 𝑄 𝑆 es mental
la oferta agregada, 𝐼 es la inversión y c. Un incremento de la riqueza domés-
𝐺 es el gasto público. tica
d. Solo a y c
I. La oferta agregada es 30 e. Ninguna de las anteriores
II. Un incremento de 10 en la inversión
incrementa el nivel de precios en 5

67 de 84
EXAMEN DE ACTUALIZACION Y SELECCIÓN BCRP
COMPILADOR MARINO ENEQUE

7. Una situación de “trampa de liqui- e. Ninguna de las anteriores


dez” 10.El producto potencial:
a. Se produce cuando la tasas de inte- a. Es cualquier punto de la frontera de
rés se elevan hacia niveles muy altos posibilidades de producción
b. Ocurre en el tramo horizontal de la b. Es el nivel de producción que man-
curva IS tiene el desempleo en su tasa natural
c. Se produce cuando la demanda por (o tasa de desempleo no aceleradora
dinero es inelástica a la tasa d interés de la inflación)
d. Reduce la efectividad con que la po- c. Es el producto base para el cálculo
lítica monetaria afecta al producto del deflactor PBI
e. Ninguna de las anteriores d. Es el nivel más alto de producción el
8. Según el modelo IS-LM, un au- ciclo económico
mento de la oferta monetaria y una e. Ninguna de las anteriores
caída en la confianza de los consumi-
dores provocará
11.En el modelo del crecimiento eco-
a. Una caída del producto y un efecto nómico de Solow (1956):
incierto en la tasa de interés
b. Un efecto incierto en el producto y un I. Un país con un bajo nivel de capital
aumento de la tasa de interés per cápita siempre presenta una tasa
c. Un efecto incierto en el producto y de crecimiento del producto per cá-
una caída en la tasa de interés pita superior a la de un país con un
d. Un aumento en el producto y una alto nivel de capital per cápita
caída en la tasa de interés II. Un incremento en la tasa de creci-
e. Una caída en el producto y un au- miento de la población incrementa el
mento en la tasa de interés capital per cápita de equilibrio en es-
tado estacionario
9. En el caso del consumo y el ahorro, III. Un incremento en la tasa de ahorro
cuál de las siguientes afirmaciones es incrementa el capital per cápita de
falsa? equilibrio en el estado estacionario
a. La existencia de restricción de liqui- IV. Un incremento en la tasa de ahorro
dez implica que el consumo depende incrementa el crecimiento del pro-
principalmente del ingreso perma- ducto de equilibrio en el estado esta-
nente cionario
b. El modelo de ciclo de vida plantea a. Sólo I y IV son verdaderas
que la propensión marginal a consu- b. Sólo II y V son verdaderas
mir depende inversamente de la c. Sólo III y IV son verdaderas
edad de la familia d. Sólo II y V son verdaderas
c. La propensión marginal al consumo e. Sólo II y III son verdaderas
del ingreso corriente es sustancial-
mente menor que la correspondiente
al ingreso permanente 12.En el modelo de demanda y oferta
d. La teoría del ingreso permanente agregada:
plantea que el ahorro responde a a. Ante un choque negativo de de-
cambios en el ingreso corriente pero manda, disminuyen los precios y au-
no del ingreso permanente menta el producto

68 de 84
EXAMEN DE ACTUALIZACION Y SELECCIÓN BCRP
COMPILADOR MARINO ENEQUE

b. Ante un choque positivo de oferta, b. Una apreciación del tipo de cambio


disminuyen los precios y aumenta el y una reducción en las exportaciones
producto netas
c. Ante un choque negativo de la oferta, c. Un aumento en la oferta monetaria
disminuyen los precios y aumenta el por entradas de capitales
producto d. Una reducción en la oferta monetaria
d. a) y c) son verdaderas por salida de capitales
e. Ninguna de las anteriores e. Movimientos endógenos en la IS por
movimiento de capitales
13.en el modelo de la demanda y
oferta agregada: 16.Considere el modelo IS-LM de eco-
a. Ante el choque positivo de demanda, nomía abierta con tipo de cambio fle-
aumentan los precios y el producto xible e imperfecta movilidad de capi-
b. Ante un choque negativa de oferta, tales. Política monetaria contractiva
disminuyen los precios y el producto genera:
c. Ante un choque negativa de oferta, a. Reducción del producto e incremento
aumentan los precios y disminuye el de la tasa de interés y el tipo de cam-
producto bio
d. a) y c) son verdaderas b. Expansión del producto, disminución
e. Ninguna de las anteriores de la tasa de interés, apreciación
cambiaria
14.Considere el modelo IS-LM de eco- c. Efecto nulo dl producto y tasa de in-
nomía abierta con tipo de cambio fle- terés combinado con depreciación
xible y perfecta movilidad de capita- cambiaria
les. Política fiscal expansiva genera: d. Contracción del producto, incre-
a. Expansión del producto e incremento mento de la tasa de interés y apre-
de la tasa de interés y el tipo de cam- ciación cambiaria
bio e. N.A.
b. Efecto nulo en el producto y tasa de
interés combinado con apreciación 17.Señala la opción verdadera. El tipo
cambiaria de cambio real se aprecia si:
c. Expansión del producto, crecimiento a. La variación de la apreciación nomi-
de las exportaciones netas y depre- nal es menor que el diferencial posi-
ciación cambiaria tivo entre la inflación mundial y la do-
d. Incremento de la tasa de interés y mestica
aumento de reservas internacionales b. El tipo de cambio nominal es fijo, la
e. N.A. inflación local es negativa y la infla-
15.En el Modelo Mundell-Fleming para ción mundial es cero
una economía pequeña y abierta, con c. La variación nominal del tipo de cam-
movilidad perfecta de capitales y tipo bio y el diferencial entre la inflación
de cambio fijo, un aumento perma- mundial y la domestica son positivos
nente en la oferta monetaria genera: d. La inflación mundial es menor que
a. Una depreciación del tipo de cambio la inflación doméstica y el tipo de
y un aumento en las exportaciones cambio nominal es fijo
netas e. N.A.

69 de 84
EXAMEN DE ACTUALIZACION Y SELECCIÓN BCRP
COMPILADOR MARINO ENEQUE

18.Señale la alternativa falsa. El incre- d. Mejora los términos de intercambio,


mento del riesgo país: incremento de las exportaciones ne-
a. Deprecia el tipo de cambio en regí- tas y expansión del producto
menes de tipo de cambio flexible e. N.A.
b. No afecta el tipo de cambio en regí-
menes de tipo de cambio fijo 21.La regla de Taylor mide:
c. Aumenta las reservas internaciona- a. La relación entre la tasa de interés y
les en regímenes de tipo de cambio el tipo de cambio
fijo b. La reacción endógena del Banco
d. Incrementa la tasa de interés domés- Central al estado de la economía.
tica en regímenes del tipo de cambio c. La relación entre las tasas de interés
fijo de corto y largo plazo
e. N.A. d. Todas son verdaderas
e. Ninguna de las anteriores
19.Con respecto a la política monetaria 22.Un aumento en la tasa de encaje en
y cambiaria, no es factible tener: moneda nacional por parte del Banco
a. Tipo de cambio fijo, control de capi- Central:
tales y un esquema de meta de infla- a. Incrementa la oferta monetaria dado
ción explicita que incrementa la velocidad de circu-
b. Tipo de cambio flexible, libre movili- lación del dinero.
dad de capitales y política monetaria b. Reduce la oferta monetaria dado que
independiente reduce la velocidad de circulación del
c. Tipo de cambio fijo, libre movilidad dinero
de capitales y un esquema de meta c. Incrementa la oferta monetaria dado
de inflación explicita que incrementa el multiplicador ban-
d. Tipo de cambio flexible, control de cario
capitales y política monetaria inde- d. Reduce la oferta monetaria dado que
pendiente reduce el multiplicador bancario
e. N.A. e. Aumenta la emisión primaria

20.En países importadores de petró- 23.Un incremento de la tasa de interés


leo, el incremento del precio interna- ocasiona:
cional del petróleo(asumiendo que el a. Un movimiento a lo largo de la curva
resto de precios de productos impor- LM
tados y de productos exportados no b. Un movimiento a lo largo de la curva
varía) genera: IS
a. Superávit de cuenta corriente combi- c. Disminución del gasto de la inversión
nado con la expansión del producto y disminución de la demanda por sal-
b. Deterioro del términos de intercam- dos reales
bio, reducción de las exportaciones d. Todas las anteriores son verdaderas
netas y la contracción del producto e. Ninguna de las anteriores
c. Deterioro de los términos de inter-
cambio, incremento del saldo en 24.Según la teoría de la paridad des-
cuenta corriente cubierta del tipo de cambio, si la tasa
de interés del país A es mayor que la
del país B, entonces:

70 de 84
EXAMEN DE ACTUALIZACION Y SELECCIÓN BCRP
COMPILADOR MARINO ENEQUE

a. El tipo de cambio bilateral de A ten-


dera a depreciarse a. Los agentes nunca se equivocan
b. El tipo de cambio de A no debería al- b. Los agentes se equivocan algunas
terarse veces
c. El tipo de cambio bilateral de A ten- c. Los agentes se equivocan, pero solo
dera a apreciarse debido a nueva información
d. El tipo de cambio multilateral de A d. Los agentes muchas veces se equi-
tendera a depreciarse vocan
e. Ninguna de las anteriores e.Ninguna de las anteriores

25.Bajo el esquema de expectativas


racionales

MICROECONOMÍA 2008

26.Indique la afirmación correcta: a. Es mayor si el bien no tiene sustitu-


a. Si X es un bien inferior y el ingreso tos
de los consumidores aumenta, la de- b. Es mayor a corto que a largo plazo
manda de X aumenta c. Indica la sensibilidad de la demanda
b. Si X es un bien normal e Y sustituto de un bien con respecto a las varia-
de X, un aumento del precio de X au- ciones del precio de sus sustitutos
menta la demanda de Y d. Es igual a I si la curva de demanda
c. Si X es un bien normal y el ingreso es una línea recta
de los consumidores disminuye, la e. Indica la sensibilidad de la demanda
demanda de X aumenta de un bien con respecto a las varia-
d. Si X es un bien normal e Y un com- ciones de su precio
plementario de X, una disminución
del precio de X aumenta la demanda 29.La elasticidad precio de un bien es
de Y mayor:
e. Si X es un bien inferior y el ingreso
a. Cuanto más y mejores sustitutos
de los consumidores aumenta, la de- tenga
manda de X no tiene variación b. En el corto plazo con respecto al
largo plazo
27.Generalmente, la función de de- c. El largo plazo con respecto al corto
manda de una mercadería que tenga plazo
pocos subtítulos es: d. Cuanto menor sea la proporción de
a. Perfectamente elástica su ingreso que gasta el consumidor
b. Inelástica en dicho bien
c. Perfectamente inelástica e. Cuanto más necesario sea el bien
d. Elástica
e. Es igual que cualquier bien 30.Cuando dos bienes son comple-
mentarios sus elasticidades- precio
28.La elasticidad precio de la demanda cruzadas son:
de un bien: a. Positivas

71 de 84
EXAMEN DE ACTUALIZACION Y SELECCIÓN BCRP
COMPILADOR MARINO ENEQUE

b. Negativas precio más bajo en el mercado cuya


c. Mayores que la unidad demanda sea más inelástica
d. Menores que la unidad c. Si lleva a cabo una discriminación
e. Nulas perfecta de precios entonces el re-
sultado no es eficiente en el sentido
31.Se tiene un bien cuya curva de de- de Pareto
manda de mercado es una recta con d. Si puede cobrar una tarifa en dos tra-
pendiente negativa. Entonces se mos elegirá un precio superior al
puede asegurar que: costo marginal
a. El bien tiene una elasticidad cons- e. Ninguna de las anteriores
tante para toda la cantidad deman-
dada 34.Señale cuál de las siguientes afir-
b. La elasticidad de demanda dismi- maciones es falsa:
nuye a medida que aumenta la canti- a. En un monopsonio la curva de costo
dad marginal de un factor es más incli-
c. El ingreso marginal es positivo para nada que la curva de oferta.
todos los precios b. Un monopsonista contrata una canti-
d. El bien no se comercia en un mer- dad ineficiente de factores de pro-
cado de competencia perfecta ducción
e. Ninguna de las anteriores c. Un monopolista cobra un precio
igual al costo marginal
32.Si la producción de un bien genera d. Es posible que gane dinero un mono-
externalidades entonces podemos polio natural que produzca un nivel
afirmar que: eficiente de producción
a. El costo social de producción es su- e. Un monopolista nunca producirá
perior al costo privado dentro del tramo inelástico de la
b. El costo social de producción es infe- curva de demanda
rior al costo privado
c. La producción del bien tiene un costo 35.Un monopolista enfrenta una fun-
social que coincide con el costo pri- ción de demanda dada por: P = 640 -
vado 3Q. los costos marginales están re-
d. La relación del costo social y el costo presentados por la función CMg = 40
privado depende del tipo de externa- + 6Q. ¿Cuál es el precio y la cantidad
lidad de equilibrio?
e. Ninguna de las anteriores. a. P= 50, Q=490
b. P= 390, Q=40
33.Señale la afirmación verdadera so- c. P= 490, Q=50
bre el comportamiento de un monopo- d. P= 150, Q=300
lista: e. P= 380, Q=60
a. Si puede cobrar precios distintos en
dos mercados diferentes, cobrara un 36.En base al siguiente grafico de una
precio más bajo en el mercado cuya función de producción.
demanda sea más elástica
b. Si puede cobrar precios distintos en
dos mercados diferentes, cobrara un

72 de 84
EXAMEN DE ACTUALIZACION Y SELECCIÓN BCRP
COMPILADOR MARINO ENEQUE

b. II y IV son falsas
c. a y b son verdaderas
d. Todas son falsas
e. Todas son verdaderas
El conjunto de producción se encuen-
tra: 39.Señale la afirmación falsa:
a. Por encima de la curva a. Cuando una firma maximiza benefi-
b. En la curva cios y es precio aceptante, los facto-
c. Debajo de la curva res que contrata son retribuidos de
d. En la curva y por debajo de la misma acuerdo a su productividad
e. Ninguna de las anteriores b. La tecnología Cobb-Douglas es un
caso especial de aquella con elastici-
37.La siguiente función de producción dad de Sustitución Constante
𝑦 = 𝐹(𝐾, 𝐿) = 𝐾 𝑎 𝐿1−𝑎 0< a <1 re- c. Cuando una firma maximiza benefi-
presenta: cios en el corto plazo, la última uni-
I. Una tecnología con rendimientos dad producida ocasiona un incre-
constantes a escala mento en el costo igual al costo pro-
II. Una función homotética medio de las unidades restantes
III. Una colección de isocuantas conve- d. A un afirma con economía de escala
xas le conviene incrementar su nivel de
producción desde el punto de vista
a. Solo I es verdadera de costos
b. Solo I y II son verdaderas e. Para una tecnología de proporciones
c. Todas son verdaderas fijas la relación marginal de Sustitu-
d. Solo III es verdadera ción Técnica se puede calcular solo
e. Ninguna es verdadera en las esquinas de las isocuantas

38.Respecto a las siguientes afirma- 40.En el corto plazo una firma deberá
ciones: abandonar su actividad:
I. Una firma que maximiza beneficios a) Cuando sus ingresos igualen a sus
necesariamente minimiza costos costos, ya que no tendría sentido
II. La curva de costos marginales de operar y no obtener ganancias
corto plazo es envolvente de todas la b) Cuando los impuestos igualen a las
curva de costos marginales de corto ganancias, pues todo el excedente
plazo deja de ser absorbido por la firma
III. El corto plazo comprende un inter- c) Cuando el precio sea menor al pro-
valo reducido de tiempo medio por unidad en el componente
IV. Una firma que minimiza los costos, variable del costo
automáticamente maximiza benefi- d) Apenas sus ingresos sean menores
cios a sus costos, pues operar bajo per-
V. La curva de costos medios de largo dida no resulta conveniente
plazo es envolvente de todas la e) Ninguna de las anteriores
curva de costos medios de corto
plazo.
a. I, I II y IV son verdaderas

73 de 84
EXAMEN DE ACTUALIZACION Y SELECCIÓN BCRP
COMPILADOR MARINO ENEQUE

74 de 84
EXAMEN DE ACTUALIZACION Y SELECCIÓN BCRP
COMPILADOR MARINO ENEQUE

BCRP EXAMEN DEL CURSO DE ACTUALIZACIÓN 2007


MACROECONOMÍA 2007
1. Bajo el enfoque IS-LM, una política ducción entre los sectores transa-
monetaria expansiva no tiene ningún bles y no transable debido a un ex-
efecto sobre el nivel del producto en cesivo gasto fiscal
el contexto de: d) Los países más ricos tienden a tener
a) Una economía pequeña y cerrada niveles de precios más altos que los
b) Una economía pequeña y abierta países menos desarrollados
con tipo de cambio flotante e) Ninguna de las anteriores
c) Una economía pequeña y abierta 4. Señale la opción verdadera:
con tipo de cambio fijo a) Los países incurren en superávit de
d) Solo a y c cuenta corriente cuando gastan más
e) Ninguna de las anteriores de su ingreso
2. Una apreciación del tipo de cambio b) Existe una relación positiva entre la
real de largo plazo en una economía cuenta corriente de una economía
pequeña y abierta estaría más proba- pequeña y abierta y la tasa de inte-
blemente asociada con: rés mundial a la que sus residentes
a) Un crecimiento sostenido de los tér- toman o concedan préstamos
minos de intercambio c) En una economía pequeña y
b) Un fuerte crecimiento de productivi- abierta, ante un choque de oferta
dad del sector transable adverso siempre se esperaría un
c) Un aumento constante de la deuda desplazamiento hacia el déficit de la
externa del país cuenta corriente
d) Solo a y b d) Solo b y c
e) Ninguna de las anteriores e) Ninguna de las anteriores
3. Señale la opción falsa: 5. Señale la opción verdadera:
a) Los países con crecimiento econó- a) El término “overshooting” del tipo de
mico más rápido tienden a experi- cambio se refiere a la respuesta más
mentar apreciaciones reales de su que proporcional que el tipo de cam-
moneda bio experimenta ante un choque mo-
netario
b) Según el esquema IS-LM, una deva-
luación del tipo de cambio tiene b) La condición de “arbitraje de tasas
efectos positivos sobre el PBI bajo de interés” establece que la tasa de
un régimen de tipo de cambio fijo interés domestica (en moneda local)
es igual a la tasa externa mas la tasa
c) La “enfermedad holandesa” se re-
de depreciación de la moneda
fiere al desplazamiento de la pro-
c) La versión relativa de la teoría de la
paridad de poder de compra implica

75 de 84
EXAMEN DE ACTUALIZACION Y SELECCIÓN BCRP
COMPILADOR MARINO ENEQUE

que la inflación interna es igual a la ii. Si la productividad del sector transa-


tasa de depreciación de la moneda ble en un país aumentaría más que
más la tasa de inflación externa en el extranjero, entonces su tipo de
d) Todas las anteriores cambio real registraría una aprecia-
e) Ninguna de las anteriores ción
6. Según la teoría de la paridad des- iii. Si la suma de las elasticidades (en el
cubierta por el tipo de cambio, si la valor absoluto) de las importaciones
tasa de interés del país A es mayor y exportaciones respecto al tipo de
que la del país B, entonces: cambio es mayor a 1, entonces, una
a) El tipo de cambio bilateral de A ten- depreciación real tipo de cambio re-
dera a depreciarse duciría el déficit comercial
b) El tipo de cambio bilateral de A ten- a) Solo (i) es verdadero
dera a apreciarse b) Solo (i) y (ii) son verdaderas
c) El tipo de cambio de A no deberá al- c) Solo (i) y (iii) son verdaderos
terarse d) (i), (ii) y (iii) son verdaderos
d) El tipo de cambio multilateral de A e) Ninguna de las anteriores
tendera a depreciarse 9. Sobre el enfoque inter-temporal de
e) Ninguna de las anteriores la cuenta corriente:
7. En una economía pequeña y i. Un choque temporal negativo en el
abierta, no se podría producir al PBI reduce la cuenta corriente en el
mismo tiempo: periodo actual
a) Tipo de cambio fijo, control de capi- ii. Un choque permanente positivo en
tales y política monetaria indepen- el PBI no afecta significativamente la
diente cuenta corriente en el periodo actual
b) Tipo de cambio flexible, control de iii. Un choque temporal positivo en los
capitales y política monetaria inde- términos de intercambio reducirá la
pendiente cuenta corriente en el periodo actual
c) Tipo de cambio fijo, libre movilidad a) Solo i es verdadero
de capitales y política monetaria in- b) Solo i y ii son verdaderos
dependiente c) Solo i y iii son verdaderos
d) Tipo de cambio flexible, libre movili- d) i, ii y iii son verdaderos
dad de capitales y política monetaria e) Ninguna de las anteriores
independiente 10.Perú es un país neto importador
e) Ninguna de las anteriores neto de combustible y productos ma-
8. Sobre el tipo de cambio real: nufacturados, y exportador de mate-
i. Si la paridad del poder de compra se rias primas. Cuál de las siguientes al-
cumple, entonces el tipo de cambio ternativas tendría un efecto positivo
real debería mantenerse constante en los términos de intercambio:

76 de 84
EXAMEN DE ACTUALIZACION Y SELECCIÓN BCRP
COMPILADOR MARINO ENEQUE

a) Un conflicto en el medio oriente que i.Al crecer la productividad marginal


irrumpa la oferta mundial del com- del capital y caer la inversión el pro-
bustible ducto de equilibrio en el mercado de
b) El Banco Central Europeo decide re- viene se incrementa
ducir sus reservas en oro ii.El precio de los bonos se incrementa
c) El sector inmobiliario de EUA (sector al subir las tasas de interés
que demanda 30% del consumo de iii.Una contracción monetaria en para-
cobre en EUA) empieza a acele- lelo con una expansión fiscal lleva a
rarse un incremento del producto y las ta-
d) BHP Billiton descubre un nuevo ya- sas de interés
cimiento de cobre en el sur del Perú iv.Si la IS es muy sensible a cambios en
e) Ninguna de las anteriores la tasa de interés luego la Política Fis-
11.De acuerdo con la hipótesis de ex- cal es más efectiva
pectativas relacionada con la curva a) Solo ii es verdadero
de rendimiento: b) Ninguna es falsa
i. Un anuncio del Banco Central en el c) ii y iv son falsas
que muestra su preocupación por d) Ninguna es verdadera
que los riesgos inflacionarios se es- e) Ninguna de las anteriores
tán incrementando produce un incre- 13.Señale la opción verdadera:
mento en la demanda por bonos so- i. Luego de una depreciación real la ba-
beranos. lanza comercial necesariamente me-
ii. Un anuncio del gobierno que para el jora.
próximo año el déficit fiscal será ma- ii. En el Modelo de Mundell-Flemming,
yor al esperado en 1 punto porcentual si el gobierno sube los impuestos
reduce las tasas de operaciones de (suma fija) el tipo de cambio se de-
mercado abierto precia. Para devolver la divisa al nivel
iii. La decisión de EEUU de elevar su anterior el banco central deberá ex-
tasa de interés no afecta a las tasas pandir la oferta monetaria
de interés locales, puesto que la eco- iii. En un modelo de economía abierta
nomía peruana se ha fortalecido y el la política fiscal es más efectiva que
riesgo país ha venido alcanzando mí- la política monetaria
nimos históricos en el último año.
iv. El efecto del gasto de gobierno so-
a) ii e iii son falsas bre el producto es mayor en una eco-
b) Todas son verdaderas nomía abierta con tipo de cambio fle-
c) i es la única verdadera xible respecto al caso de una econo-
d) i y ii son verdaderas mía cerrada
e) Ninguna de las anteriores a) iii es la única falsa
12.Señale la opción verdadera: b) ii es la única verdadera
c) iv es la única verdadera

77 de 84
EXAMEN DE ACTUALIZACION Y SELECCIÓN BCRP
COMPILADOR MARINO ENEQUE

d) i y iii son verdaderas e) Ninguna de las anteriores


e) Ninguna de las anteriores 15.La inversión es más volátil que el
14.Considere el siguiente modelo: consumo porque:
C = 50 + 0.6 (Y-T) a) Las familias son adversas al riesgo
I = 10 + 0.1Y-i b) Las empresas enfrentan más incer-
G = 100 tidumbres que las familias
X=M=0 c) Las familias tienen mayor acceso al
Donde C es el consumo privado, Y es crédito que las familias
el ingreso, T es el impuesto de suma d) Los impuestos son más altos a la in-
fija, i es el interés de 5%, G es el gasto versión
público y X y M son exportaciones e im- e) Ninguna de las anteriores
portaciones respectivamente. Asuma 16.De acuerdo con la hipótesis del in-
inicialmente que el presupuesto público greso permanente para el consumo.
esta balanceado. Señale la proposición En una economía cerrada, ante un
falsa. aumento en el ingreso futuro de las
i.Un aumento de 10 en el Gasto Pu- familias, la tasa de interés real en el
blico (G) (asumiendo que el presu- corto plazo.
puesto esta balanceado) lleva a un a) Aumentará más si el aumento es
aumento de 13.3 del ingreso del equi- permanente
librio (Y) b) Caerá si el aumento es transitorio
ii.Si i=10 el multiplicador asciende a c) No se modifica
10.33 y permanece constante en el d) Caerá si el aumento es transitorio, y
tiempo la inversión es inelástica en el corto
iii.Si el impuesto toma la forma de una plazo
proporción del ingreso (T=tY), el e) Ninguna de las anteriores
efecto de un aumento de 10 en el
17.La brecha producto se define como:
Gasto Publico (G) en el ingreso de
a) El nivel del producto por encima del
equilibrio (Y) sería mayor que si fuera
producto potencial
de suma fija, debido a que crece el
b) El nivel del producto que equilibra el
multiplicador
mercado de bienes
iv.Si el impuesto toma la forma de una
c) El nivel del producto que genera pre-
proporción del ingreso (T=tY), el pre-
siones de demanda
supuesto permanece balanceado,
pero el efecto del gasto es menor so- d) El nivel de producto consiste en in-
bre el ingreso flación cero
a) Solo i es la verdadera e) Ninguna de las anteriores
b) Solo i y iii es la verdadera 18.Bajo el esquema de expectativas
c) Solo ii y iv es la verdadera racionales:
d) Solo i, ii y iv es la verdadera a) Los agentes nunca se equivocan

78 de 84
EXAMEN DE ACTUALIZACION Y SELECCIÓN BCRP
COMPILADOR MARINO ENEQUE

b) Los agentes se equivocan, pero solo a) La relación entre tasas de interés y


debido a nueva información el tipo de cambio
c) Los agentes se equivocan algunas b) La reacción endógena del Banco
veces Central al estado de economía
d) Los agentes muchas veces se equi- c) La relación entre tasas de interés de
vocan corto y largo plazo
e) Ninguna de las anteriores d) Todas son verdaderas
19.En el modelo de demanda y oferta e) Ninguna de las anteriores
agregada: 23.Según el modelo de Solow. Un au-
a) Ante un choque positivo de de- mento en la tasa de ahorro de la eco-
manda, aumentan los precios y el nomía:
producto a) Aumenta el producto por trabajador
b) Ante un choque negativa de oferta, en el largo plazo
disminuyen los precios y el producto b) Un menor nivel de consumo en el
c) Ante un choque negativa de oferta, largo plazo
aumentan los precios y disminuye el c) Mayor productividad en el largo
producto plazo
d) a y c son verdaderas d) Mayor tasa de crecimiento del pro-
e) Ninguna de las anteriores ducto por trabajador en el largo
20.De acuerdo con la paridad no cu- plazo
bierta de tasas de interés ante un au- e) Ninguna de las anteriores.
mento de la tasa de interés interna- 24.El dinero es:
cional: a) Cualquier objeto aceptado univer-
a) El tipo de cambio se deprecia salmente como medio de pago
b) El tipo de cambia se aprecia b) Únicamente billetes y monedas en
c) La demanda por activos en moneda circulación
doméstica disminuye c) Únicamente los billetes y monedas
d) a y c son verdaderas emitidos por el banco central
e) Ninguna de las anteriores. d) Es oro y plata
21.De acuerdo con la curva de Phillips e) Ninguna de las anteriores
existe una relación positiva entre: 25.El residuo de Solow mide:
a) La inflación y la tasa de empleo a) La productividad del trabajo
b) La inflación y los costos marginales b) La productividad de capital
c) La inflación y la brecha producto c) La mejora en la tecnología
d) La inflación y las tasas de interés d) La productividad de todos los facto-
e) Ninguna de las anteriores res de producción
22.La regla de Taylor mide: e) Ninguna de las anteriores

79 de 84
EXAMEN DE ACTUALIZACION Y SELECCIÓN BCRP
COMPILADOR MARINO ENEQUE

MICROECONOMÍA 2007
26.Una persona no puede establecer 10 unidades la cantidad disminuya en
sus preferencias entre dos canastas 3 unidades. Si la pendiente de la fun-
de consumo distinta A y B. En este ción de demanda indica que el precio
caso, decimos que sus preferencias aumente en 10 unidades al disminuir
no son: la cantidad en 1 unidad, señale la afir-
a) Transitivas mación verdadera:
b) Reflexivas a) El efecto ingreso será negativo indi-
cando que se trata de un bien infe-
c) Completas
rior
d) Ordinales
b) El efecto ingreso será positivo indi-
e) Ninguna de las anteriores
cando que se trata de un bien nor-
27.En un mercado en competencia mal
perfecta, no es correcto que: c) Para que la pendiente de la función
a) En equilibrio de largo plazo, las dife- de demanda sea negativa, obligato-
rentes empresas terminan operando riamente el efecto ingreso debe ser
cada una en el punto mínimo de su positivo
costo medio de largo plazo d) El efecto ingreso siempre es nega-
b) La oferta de cada empresa, en el tivo en una función de demanda de-
corto plazo, es una curva de costo creciente
marginal, a partir del costo mínimo e) Ninguna de las anteriores
de su costo medio variable
29.A una persona que le gusta el café
c) El número de empresas operando
y lo toma con azúcar. Consume men-
en equilibrio de largo plazo es deter-
sualmente 2 Kg. de café y 3 kg. de
minado por la magnitud de la de-
azúcar. Si “X” representa el café e
manda y la cantidad ofertada por
“Y” el azúcar ¿cuál de estas funcio-
cada empresa en el punto mínimo
nes representa sus preferencias?
del costo medio de largo plazo
a) U1= min {x/2,y/3}
d) Cada empresa elige ofertar en el
b) U2= 2*x+3*y
punto en el cual el precio de mer-
cado prevaleciente es igual a su c) U3= min {x/3,y/2}
costo marginal d) U4= x(1/2) y (1/3)
e) Ninguna de las anteriores e) Ninguna de las anteriores.
28.Partiendo de una situación de equi- 30.Un consumidor con una función de
librio (donde la cantidad y el precio de utilidad U=10+log (x)+2log (y) maxi-
equilibrio son 20 y 100 respectiva- miza actualmente consumiendo la
mente), el efecto sustitución hace canasta (10,15). ¿Cuál de las si-
que ante un aumento en el precio de guientes afirmaciones es cierta?

80 de 84
EXAMEN DE ACTUALIZACION Y SELECCIÓN BCRP
COMPILADOR MARINO ENEQUE

a) La relación de precios es d) (1+i)m1+m2


(Px/Py)=3/2 e) Ninguna de la anteriores
b) La relación de precios es 33.En el contexto de un modelo inter-
(Px/Py)=2/3 temporal y partiendo de una situación
c) La relación de precios es donde el individuo inicialmente estu-
(Px/Py)=3/4 viese ahorrando, el gobierno decide
d) La relación de precios es aplicar un impuesto sobre el ingreso
(Px/Py)=4/3. futuro. El efecto de esta política será:
e) Ninguna de las anteriores a) Un desplazamiento paralelo de la
31.En un modelo de competidores de restricción presupuestaria hacia el
Bertrand, si la demanda viene dada origen
por la ecuación P=100-2(q1+q2) y la b) Si los bienes de consumo en el pe-
empresa 1 establece un precio P1=20 riodo 1 y 2 fuesen normales, dismi-
y la empresa 2 un precio P2=25. En nuirían las cantidades consumidas.
este caso : c) Aumentaría el ahorro del individuo
a) La empresa 1 venderá 40 unidades d) Todas las respuestas anteriores son
y la empresa 2 no venderá ninguna correctas
unidad. e) Ninguna de las anteriores
b) Tanto la empresa 1 como la 2 ven- 34.Suponga que un monopolista puro
derán 20 unidades cada una está produciendo en un punto donde
c) La empresa 1 no venderá ninguna la elasticidad E=-3. Si el costo margi-
cantidad y la empresa 2vendera 40 nal es constante e igual a 10, el pre-
unidades cio al que vende el producto es igual
d) La empresa 1 no venderá ninguna a:
unidad y la empresa 2 venderá 37.5 a) 30
unidades b) 25
e) Ninguna de las anteriores c) 15
32.Suponga que un individuo consuma d) No es posible calcular el precio sin
C1 en el periodo 1 y C2 en el periodo conocer la curva de demanda
2, mientras que su ingreso en m1 y e) Ninguna de las anteriores
m2 en cada periodo y la tasa de inte-
35.Considere el caso de empresas re-
rés es i. Entonces si en el periodo 1
lacionadas (con un único propietario),
no consume nada, la cantidad má-
donde una de ellas es la empresa
xima que puede consumir en el pe-
matriz, que opera en un mercado
riodo 2 es:
competitivo y una empresa subsidia-
a) C1/(1+i)
ria, que es una empresa monopólica
b) m2/(1+i) en su industria, y que provee de insu-
c) m1 + m2/(1+i) mos a la matriz.

81 de 84
EXAMEN DE ACTUALIZACION Y SELECCIÓN BCRP
COMPILADOR MARINO ENEQUE

Se conoce la siguiente información: b) En un monopolio perfecto, el mono-


Costos totales de la subsidiaria: polista opera en el tramo inelástico
CT = 10 + 5Q + 0.5Q2 de la demanda, debido a que si la
elasticidad fuera mayor a 1, el in-
Demanda de la matriz:
greso marginal no podría interceptar
Pm = 50 – 2Qm al costo marginal
Demanda del resto de empresas: c) La discriminación de precios es po-
Pr = 100-Qr sible en cada uno de los mercados
Determine (PR, Pm, Qr, Qm): son diferentes
a) (70, 40, 30, 5) d) Si la demanda tiene una elasticidad
b) (205/3, 20, 95/3, 15) constante e igual a uno, el monopo-
c) (1280/21,1280/21,235/7,235/7) lista puede escoger entre diferentes
puntos de equilibrios óptimos
d) (0, 20, 0, 15)
e) Ninguna de las anteriores
e) Ninguna de las anteriores.
38.Señale la proporción acorde con la
36.Indique la alternativa incorrecta:
ley de los mercados de Walras:
a) Cuando existen costos de produc-
a) En una economía con un mercado,
ción fijos, el costo marginal es cons-
si n-1 de ellos están en desequili-
tante y el costo medio es decre-
brio, entonces el n-ésimo mercado
ciente y asintótico al costo marginal
también lo estará
b) La presencia de economías de es-
b) El valor del exceso de demanda
cala lleva que la curva de costo me-
agregada es idénticamente igual
dio de largo plazo sea decreciente
acero
c) El ingreso marginal es horizontal
c) Esta ley se verifica solamente para
para el caso de precio variable, y es
aquellas situaciones de equilibrio de
decreciente para el caso de precio
los mercados
fijo
d) Todas las proposiciones son verda-
d) Si el costo total fuera lineal y partiera
deras
del origen, entonces el costo medio
seria idéntico al costo marginal e) Todas las proposiciones son falsas

e) Ninguna de las anteriores 39.Si la curva de oferta de trabajo tiene


pendiente negativa, entonces el
37.Indique la alternativa correcta:
efecto situación de un aumento sala-
a) En un mercado de competencia per- rial:
fecta, si existen deseconomías de
a) Es dominado por el efecto ingreso
escala y todas las empresas son
cuando el salario es alto
iguales, la curva de oferta de largo
plazo será de pendiente negativa b) Domina el efecto ingreso cuando el
salario es alto

82 de 84
EXAMEN DE ACTUALIZACION Y SELECCIÓN BCRP
COMPILADOR MARINO ENEQUE

c) Opera en la misma dirección que el conocidas por tener elasticidad de


efecto ingreso sustitución:
d) Se vuelve más importante conforme a) Constante y cero respectivamente
transcurre el tiempo b) Cero y constante respectivamente
e) Es inconsistente con el comporta- c) Infinita y cero respectivamente
miento maximizador de los trabaja- d) No se puede determinar para la pri-
dores mera y cero para la segunda
40.Las funciones de producción Cobb- e) Ninguna de las anteriores
Douglas y de Coeficientes Fijos son

83 de 84
EXAMEN DE ACTUALIZACION Y SELECCIÓN BCRP
COMPILADOR MARINO ENEQUE

84 de 84

También podría gustarte